You are on page 1of 97

PH YSI CS

DPP
DAILY PRACTICE PROBLEMS

TARGET : JEE(MAIN + ADVANCED)


ST

NO. 1
HEAT AND THERMODYNAMICS
TOPIC: HEAT TRANSFER

1. A uniform slab of dimension 10cm × 10cm × 1cm is kept between two heat reservoirs at
temperatures 10ºC and 90ºC. The larger surface areas touch the reservoirs. The thermal
conductivity of the material is 0.80 W/m–ºC. Find the amount of heat flowing through the slab
per second.
2. One end of a steel rod (K = 42 J/m–s–ºC) of length 1.0 m is kept in ice at 0ºC and the other
end is kept in boiling water at 100ºC. The area of cross–section of the rod is 0.04cm2.
Assuming no heat loss to the atmosphere, find the mass of the ice melting per second. Latent
heat of fusion of ice = 3.36 × 105 J/kg.
3. A rod CD of thermal resistance 5.0 K/W is joined at the middle of an identical rod AB as
shown in figure. The ends A, B and D are maintained at 100ºC, 0ºC and 25ºC respectively.
Find the heat current in CD.

4. A semicircular rod is joined at its ends to a straight rod of the same material and same cross-
sectional area. The straight rod forms a diameter of the other rod. The junctions are
maintained at different temperatures. Find the ratio of the heat transferred through a cross-
section of the semicircular rod to the heat transferred through a cross-section of the straight
rod in a given time.

5. Three slabs of same surface area but different conductivities k1, k2, k3 and different thickness
t1, t2, t3 are placed in close contact. After steady state this combination behaves as a single
slab. Find its effective thermal conductivity.

DOWNLOAD OUR APP & GET : 353, Rajeev Gandhi Nagar, Instrumentation Limited Colony, Kota, Rajasthan 324005

UNLIMITED PRACTICE FOR FREE : www.atpstar.com PAGE NO.-1


6. A wall has two layers A and B, each made of different material. Both the layers have the same
thickness. The thermal conductivity for A is twice that of B. Under steady state, the
temperature difference across the whole wall is 36°C. Then the temperature difference across
the layer A is
(A) 6°C (B) 12°C (C) 18°C (D) 24°C

7. Two metal cubes with 3 cm-edges of copper and aluminium are arranged as shown in figure
(assume no loss of heat from open surfaces)
(KCU =385 W/m-K, KAL = 209 W/m-K)
(a) The total thermal current from one reservoir to the other is :

(A) 1.42 × 103 W (B) 2.53 × 103 W


(C) 1.53 × 104 W (D) 2.53 × 104 W

(b) The ratio of the thermal current carried by the copper cube to that carried by the
aluminium cube is : –
(A) 1.79 (B) 1.69 (C) 1.54 (D) 1.84

8. A wall consists of alternating blocks with length ‘d’ and coefficient of thermal conductivity k1
and k2. The cross sectional area of the blocks are the same. The equivalent coefficient of
thermal conductivity of the wall between left and right is :-

( K1  K2 ) K1 K 2 2 K1 K 2
(A) K1 + K2 (B) (C) (D)
2 K1  K 2 K1  K 2

DOWNLOAD OUR APP & GET : 353, Rajeev Gandhi Nagar, Instrumentation Limited Colony, Kota, Rajasthan 324005

UNLIMITED PRACTICE FOR FREE : www.atpstar.com PAGE NO.-2


PH YSI CS

DPP
DAILY PRACTICE PROBLEMS

TARGET : JEE(MAIN + ADVANCED)


ST

NO. 2
HEAT AND THERMODYNAMICS
TOPIC: HEAT TRANSFER

1. A boiler is made of a copper plate 2.4 mm thick with an inside coating of a 0.2 mm thick layer
of tin. The surface area exposed to gases at 700°C is 400 cm2. The amount of steam that
could be generated per hour at atmospheric pressure is (Kcu = 0.9 and Ktin = 0.15 cal/cm/s/°C
and Lsteam = 540 cal/g)
(A) 5000 Kg (B) 1000 kg (C) 4000 kg (D) 200 kg

2. A lake surface is exposed to an atmosphere where the temperature is < 0°C. If the thickness
of the ice layer formed on the surface grows from 2 cm to 4 cm in 1 hour, The atmospheric
temperature, Ta will be-
(Thermal conductivity of ice K = 4 x 10-3 cal/cm/s/°C; density of ice = 0.9 gm/cc. Latent heat of
fusion of ice = 80 cal/gm. Neglect the change of density during the state change. Assume that
the water below the ice has 0º temperature every where)

(A) – 20 °C (B) 0 °C (C) – 30 °C (D) – 15 °C

3. A hollow metallic sphere of radius 20 cm surrounds a concentric metallic sphere of radius 5


cm. The space between the two spheres is filled with a nonmetallic material. The inner and
outer spheres are maintained at 50°C and 10°C respectively and it is found that 160 Joule of
heat passes radially from the inner sphere to the outer sphere per second. Find the thermal
conductivity of the material between the spheres.

4. A hollow tube has a length , inner radius R1 and outer radius R2. The material has thermal

conductivity K. Find the heat flowing through the walls of the tube per second if the inside of
the tube is maintained at temperature T1 and the outside is maintained at T2 [assume T2 > T1]

DOWNLOAD OUR APP & GET : 353, Rajeev Gandhi Nagar, Instrumentation Limited Colony, Kota, Rajasthan 324005

UNLIMITED PRACTICE FOR FREE : www.atpstar.com PAGE NO.-3


5. Heat flows radially outward through a spherical shell of outside radius R2 and inner radius R1.
The temperature of inner surface of shell is 1 and that of outer is . The radial distance from
centre of shell where the temperature is just half way between 1 and 2 is :
R1  R2 R1 R 2 2 R1 R2 R2
(A) (B) (C) (D) R1 +
2 R1  R 2 R1  R2 2

6. A metal rod of cross-sectional area 1.0 cm2 is being heated at one end. At one time, the
temperature gradient is 5.0°C/cm at cross-section A and is 2.6 °C/cm at cross-section B.
Calculate the rate at which the temperature is increasing in the part AB of the rod. The heat
capacity of the part AB = 0.40 J/°C, thermal conductivity of the material of the rod = 200 W/m–
°C. Neglect any loss of heat to the atmosphere.

7. A metallic sphere having radius 0.08 m and mass m = 10kg is heated to a temperature of
227°C and suspended inside a box whose walls are at a temperature of 27°C. The maximum
rate at which its temperature will fall is :-
(Take e = 1, Stefan’s constant  = 5.8 x 10-8 Wm-2 K-4 and specific heat of the metal s = 90
cal/kg/deg J = 4.2 Joules/Calorie)
(A) .055 °C/sec (B) .066 °C/sec
(C) .044 °C/sec (D) 0.03 °C/sec

8. A solid spherical black body of radius r and uniform mass distribution is in free space. It emits
power ‘P’ and its rate of colling is R then
(A) R P  r2 (B) R P  r (C) R P  1/r2 (D) R P 

DOWNLOAD OUR APP & GET : 353, Rajeev Gandhi Nagar, Instrumentation Limited Colony, Kota, Rajasthan 324005

UNLIMITED PRACTICE FOR FREE : www.atpstar.com PAGE NO.-4


PH YSI CS

DPP
DAILY PRACTICE PROBLEMS

TARGET : JEE(MAIN + ADVANCED)


ST

NO. 3
HEAT AND THERMODYNAMICS
TOPIC: HEAT TRANSFER

1. Three separate segments of equal area A1, A2 and A3 are shown in the energy distribution
curve of a blackbody radiation. If n1, n2 and n3 are number of photons emitted per unit time
corresponding to each area segment respectively then:
E

A1 A2 A3 
(A) n2 > n1 > n3 (B) n3 > n1 > n2 (C) n1 = n2 = n3 (D) n3 > n2 > n1
2. When q1 joules of radiation is incident on a body it reflects and transmits total of q2 joules.
Find the emissivity of the body.
3. A blackbody of surface area 1 cm2 is placed inside an enclosure. The enclosure has a
constant temperature 27ºC and the blackbody is maintained at 327ºC by heating it electrically.
What electric power is needed to maintain the temperature?  = 6.0 × 10–8 W/m2 –K4.
4. Estimate the temperature at which a body may appear blue or red. The values of mean for
these are 5000 Å and 7500Å respectively. [ Given Wein’s constant b = 0.3 cm K ]
5. The temperature of a hot liquid in a container of negligible heat capacity falls at the rate of 3
K/min due to heat emission to the surroundings, just before it begins to solidify. The
temperature then remains constant for 30 min, by the time the liquid has all solidified. Find the
ratio of specific heat capacity of liquid to specific latent heat of fusion.
6. The earth receives at its surface radiation from the sun at the rate of 1400 Wm-2. The distance
of the centre of the sun from the surface of the earth is 1.5 × 1011 m and the radius of the sun
is 7 × 108 m. Treating the sun as a black body calculate temperature of sun.
7. A solid copper sphere (density  and specific heat c) of radius r at an initial temperature 200 K
is suspended inside a chamber whose walls are at almost 0 K. Calculte the time required for
the temperature of the sphere to drop to 100 K. (Assume sphere as a black body)
8. Which of the law can be understood in terms of Stefan's law
(A) Wien’s displacement law (B) Kirchoff’s law
(C) Newton’s law of cooling (D) Planck’s law

DOWNLOAD OUR APP & GET : 353, Rajeev Gandhi Nagar, Instrumentation Limited Colony, Kota, Rajasthan 324005

UNLIMITED PRACTICE FOR FREE : www.atpstar.com PAGE NO.-5


PH YSI CS

DPP
DAILY PRACTICE PROBLEMS

TARGET : JEE(MAIN + ADVANCED)


ST

NO. 4
HEAT AND THERMODYNAMICS
TOPIC: HEAT TRANSFER

1. A hot liquid is kept in a big room. According to Newton's law of cooling rate of cooling of liquid
(represented as y) is plotted against its temperature T. Which of the following curves may
represent the plot ?

(A) (B) (C) (D)

2. A liquid cools from 70ºC to 60ºC in 5 minutes. Find the time in which it will further cool down to
50 ºC, if its surrounding is held at a constant temperature of 30ºC.

3. Two identical square rods of metal are welded end to end as shown in figure (a). Assume that
10 cal of heat flows through the rods in 2 min. Now the rods are welded as shown in figure,
(b). The time it would take for 10 cal to flow through the rods now, is

100°C
0°C

(a)

(A) 0.75 min (B) 0.5 min (C) 1.5 min (D) 1 min

4. Three metal rods made of copper, aluminium and brass, each 20 cm long and 4 cm in
diameter, are placed end to end with aluminium between the other two. The free ends of
copper and brass are maintained at 100 and 0°C respectively. Assume that the thermal
conductivity of copper is twice that of aluminium and four times that of brass. The
approximately equilibrium temperatures of the copper-aluminium and aluminium-brass
junctions are respectively.

(A) 68 °C and 75 °C (B) 75 °C and 68 °C (C) 57 °C and 86 °C (D) 86 °C and 57 °C

DOWNLOAD OUR APP & GET : 353, Rajeev Gandhi Nagar, Instrumentation Limited Colony, Kota, Rajasthan 324005

UNLIMITED PRACTICE FOR FREE : www.atpstar.com PAGE NO.-6


5. A closed cubical box is made of a perfectly insulating material walls of thickness 8 cm and the
only way for heat to enter or leave the box is through two solid metallic cylindrical plugs, each
of cross-sectional area 12 cm2 and length 8 cm, fixed in the opposite walls of the box. The
outer surface A on one plug is maintained at 100°C while the outer surface B of the other plug
is maintained at 4°C. The thermal conductivity of the material of each plug is 0.5 cal/°C/cm. A
source of energy generating 36 cal/s is enclosed inside the box. Assuming the temperature to
be the same at all points on the inner surface, the equilibrium temperature of the inner surface
of the box is

A Source B
100°C of 4°C
Energy

8 cm
Insulating
Walls
(A) 62 °C (B) 46 °C (C) 76 °C (D) 52 °C

6. Two models of a windowpane are made. In one model, two identical glass panes of thickness
3 mm are separated with an air gap of 3 mm. This composite system is fixed in the window of
a room. The other model consists of a single glass pane of thickness 6 mm, the temperature
difference being the same as for the first model. The ratio of the heat flow for the double pane
to that for the single pane is
(Kglass = 2.5 × 10–4 cal/s.m. °C and Kair = 6.2 × 10–6 cal/s.m.°C)
(A) 1/20 (B) 1/70 (C) 31/1312 (D) 31/656

7. Heat is flowing through two cylindrical rods made of same materials whose ends are
maintained at similar temperatures. If diameters of the rods are in ratio 1 : 2 and lengths in
ratio 2 : 1, then the ratio of thermal current through them in steady state is :
(A) 1 : 8 (B) 1 : 4 (C) 1 : 6 (D) 4 : 1

8. The ends of a metre stick are maintained at 100ºC and 0ºC. One end of a rod is maintained at
25ºC. Where should its other end be touched on the metre stick so that there is no heat
current in the rod in steady state?
(A) 25 cm from the hot end (B) 40 cm from the cold end
(C) 25 cm from the cold end (D) 60 cm from the cold end

DOWNLOAD OUR APP & GET : 353, Rajeev Gandhi Nagar, Instrumentation Limited Colony, Kota, Rajasthan 324005

UNLIMITED PRACTICE FOR FREE : www.atpstar.com PAGE NO.-7


PH YSI CS

DPP
DAILY PRACTICE PROBLEMS

TARGET : JEE(MAIN + ADVANCED)


ST

NO. 5
HEAT AND THERMODYNAMICS
TOPIC: HEAT TRANSFER

1. A spherical solid black body of radius ‘r’ radiates power ‘H’ and its rate of cooling is ‘C’. If
density is constant then which of the following is/are true.
1
(A) H  r and c  r2 (B) H  r2 and c 
r
1
(C) H  r and c  (D) H  r2 and c  r2
r2
2. The earth is getting energy from the sun whose surface temperature is Ts and radius is R. Let
the radius of the earth the r and the distance from the sun be d. Assume the earth and the sun
both to behave as perfect black bodies and the earth is in thermal equilibrium at a constant
temperature Te. Therefore, the temperature Ts of the sun is xTe where x is
2d 2R 4d d
(A) (B) (C) (D)
R r r r
3. The plots of intensity vs. wavelength for three black bodies at temperatures T1, T2 and T3
respectively are as shown. Their temperatures are such that-
 T3
T2
T1


(A) T1 > T2 > T3 (B) T1 > T3 > T2
(C) T2 > T3 > T1 (D) T3 > T2 > T1
4. The temperature of bodies X and Y vary with time as shown in the figure. If emissivity of
bodies X and Y are eX & eY and absorptive powers are AX and AY, (assume other conditions
are identical for both):then:

(A) eY > eX , AY > AX (B) eY < eX , AY < AX


(C) eY > eX , AY < AX (D) eY < eX , AY > AX

DOWNLOAD OUR APP & GET : 353, Rajeev Gandhi Nagar, Instrumentation Limited Colony, Kota, Rajasthan 324005

UNLIMITED PRACTICE FOR FREE : www.atpstar.com PAGE NO.-8


5. Three discs of same material A, B, C of radii 2 cm, 4 cm and 6 cm respectively are coated
with carbon black. Their wavelengths corresponding to maximum spectral radiancy are 300,
400 and 500 nm respectively then maximum power will be emitted by

(A) A (B) B (C) C (D) same for all


6. Three graphs marked as 1, 2, 3 representing the variation of maximum emissive power and
wavelength of radiation of the sun, a welding arc and a tungsten filament. Which of the
following combination is correct

(A) 1- bulb, 2  welding arc, 3  sun (B) 2- bulb, 3  welding arc, 1  sun
(C) 3- bulb, 1  welding arc, 2  sun (D) 2- bulb, 1  welding arc, 3  sun

DOWNLOAD OUR APP & GET : 353, Rajeev Gandhi Nagar, Instrumentation Limited Colony, Kota, Rajasthan 324005

UNLIMITED PRACTICE FOR FREE : www.atpstar.com PAGE NO.-9


PH YSI CS

DPP
DAILY PRACTICE PROBLEMS

TARGET : JEE(MAIN + ADVANCED)


ST

NO. 6
HEAT AND THERMODYNAMICS
TOPIC: CALORIMETRY

1. The amount of heat required to change the state of 1 kg of substance at constant temperature
is called

(A) kilocal (B) calorie (C) specific heat (D) latent heat

2. The water equivalent of a 400 g copper calorimeter (specific heat = 0.1 cal/gºC)

(A) 40 g (B) 4000 g (C) 200 g (D) 4 g

3. Heat required to convert 1 g of ice at 0ºC into steam at 100ºC is

(A) 100 cal (B) 0.01 cal/ºC (C) 720 cal (D) 1 kilocal

4. The thermal capacity of 40 g of aluminium (specific heat = 0.2 cal/gmºC)

(A) 40 cal/ºC (B) 160 cal/ºC (C) 200 cal/ºC (D) 8 cal/ºC

5. The boiling water is changing into steam. Under this condition, the specific heat of water is

(A) zero (B) one (C) infinite (D) less than one

6. One kg of ice at 0ºC is mixed with 1 kg of water at 10ºC. The resulting temperature will be

(A) between 0ºC and 10ºC (B) 0ºC

(C) less than 0ºC (D) greater than 0ºC

7. If 10g of ice at 0ºC is mixed with 10g of water at 40ºC, the final mass of water in the mixture is

(A) 10 g (B) 15 g (C) 18 g (D) 20 g

8. 540 g of ice at 0ºC is mixed with 540 g of water at 80ºC. The final temperature of the mixure is

(A) 0ºC (B) 40ºC (C) 80ºC (D) less than 0ºC

9. Steam at 100ºC is passed into 2.0 kg of water contained in a calorimeter of water equivalent
0.02 kg at 15ºC till the temperature of the calorimeter and its contents rise to 90ºC. The mass
of steam condensed in kg is

(A) 0.301 (B) 0.280 (C) 0.60 (D) 0.02

DOWNLOAD OUR APP & GET : 353, Rajeev Gandhi Nagar, Instrumentation Limited Colony, Kota, Rajasthan 324005

UNLIMITED PRACTICE FOR FREE : www.atpstar.com PAGE NO.-10


PH YSI CS

DPP
DAILY PRACTICE PROBLEMS

TARGET : JEE(MAIN + ADVANCED)


ST

NO. 7
HEAT AND THERMODYNAMICS
TOPIC: THERMAL EXPANSION

1. Two large holes are cut in a metal sheet. If this is heated, distances AB
and BC, (as shown)
(A) both will increase (B) both will decrease
(C) AB increases, BC decreases (D) AB decreases, BC increases
2. Expansion during heating (generally)
(A) occurs only in a solid
(B) increases the density of the material
(C) decreases the density of the material
(D) occurs at the same rate for all liquids and solids.
3. Two bars of copper having same length but unequal diameter are heated to the same
temperature. The change in length will be -
(A) More in thinner bar
(B) More in thicker bar
(C) Same for both the bars
(D) Determined by the ratio of length and diameter of the bars
4. A motallic bar is heated from 0ºC to 100ºC. The coefficient of linear expansion is 10–5K–1.
What will be the percentage increase in length
(A) 0.01% (B) 0.1% (C) 1% (D) 10%
–5
5. A pendulum clock has an iron pendulum 1m long (iron = 10 /ºC). If the temperature rises by
10ºC, the clock
(A) Will lose 8 seconds per day (B) Will lose 4.32 seconds per day
(C) Will gain 8 seconds per day (D) Will gain 4.32 seconds per day
6. Two rods of lengths 1 and 2 are made of materials whose coefficient of linear expansions are
1 are 2. If the difference between two lengths is independent of temperature -
1 1 1 2 12 22
(A)  =  (B)  =  (C) 221 = 12 2 (D)  = 
2 2 2 1 1 2

7. If , ,  are linear, superficial and cubical expansivity of a solid, then -


(A)  :  :  = 1 : 2 : 3 (B)  :  :  = 3 : 2 : 1 (C)  :  :  = 2 : 3 : 1 (D)  :  :  = 3 : 1 : 3
8. The coefficient of linear expansion of steel and brass are 11 × 10–6/ºC and 19 × 10–6/ºC
respectively. If their difference in lengths at all temperatures has to be kept constant at 30cm,
their lengths at 0ºC should be -
(A) 71.25 cm and 41.25 cm (B) 82 cm and 52 cm
(C) 92 cm and 62 cm (D) 62.25 cm and 32.25 cm

DOWNLOAD OUR APP & GET : 353, Rajeev Gandhi Nagar, Instrumentation Limited Colony, Kota, Rajasthan 324005

UNLIMITED PRACTICE FOR FREE : www.atpstar.com PAGE NO.-11


9. A solid ball of metal has a spherical cavity inside it. If the ball is heated, the volume of the
cavity will -
(A) Increase (B) Decrease
(C) Remains unchanged (D) Have its shape changed
10. If the length of a cylinder on heating increases by 2%, the area of its base will increase by-
(A) 0.5% (B) 2% (C) 1% (D) 4%
11. A uniform metal rod is used as a bar pendulum. If the room temperature rises by 10ºC, and
the coefficient of linear expansion of the metal of the rod is 2 × 10–6perºC, the period of the
pendulum will have percentage increase of
(A) – 2 × 10–3 (B) – 1 × 10–3 (C) 2 × 10–3 (D) 1 × 10–3
12. The volume of a solid decreases by 0.6% when it is cooled through 50ºC. Its coefficient of
linear expansion is -
(A) 4 × 10–6K (B) 5 × 10–5K (C) 6 × 104K (D) 4 × 10–5K
13. Which of the following curve represent variation of density of water with temperature best -
density

density

density

density
(A) (B) (C) (D)
temp temp temp temp

14. A rectangular block is heated from 0ºC to 100ºC. The percentage increase in its length is
0.10% What will be the percentage increase in it volume?
(A) 0.03 % (B) 0.10% (C) 0.30% (D) none of these

15. A thin copper wire of length increases in length by 1% when heated from 0ºC to 100ºC. If a
then cooper plate of area 2 ×  is heated from 0ºC to 100ºC, the percentage increase in its
area will be
(A) 1% (B) 2% (C) 3% (D) 4%

DOWNLOAD OUR APP & GET : 353, Rajeev Gandhi Nagar, Instrumentation Limited Colony, Kota, Rajasthan 324005

UNLIMITED PRACTICE FOR FREE : www.atpstar.com PAGE NO.-12


PH YSI CS

DPP DAILY PRACTICE PROBLEMS

TARGET : JEE(MAIN + ADVANCED)


ST

NO. 8
HEAT AND THERMODYNAMICS
TOPIC: KINETIC THEORY OF GASES

1. When an ideal gas is compressed isothermally then its pressure increases because :
(A) its potential energy decreases
(B) its kinetic energy increases and molecules move apart
(C) its number of collisions per unit area with walls of container increases
(D) molecular energy increases
2. Which of the following quantities is zero on an average for the molecules of an ideal gas in
equilibrium ?
(A) kinetic energy (B) momentum (C) density (D) speed
3. The average momentum of a molecule in a sample of an ideal gas depends on
(A) temperature (B) number of moles (C) volume (D) none of these
4. A gas behaves more closely as an ideal gas at
(A) low pressure and low temperature (B) low pressure and high temperature
(C) high pressure and low temperature (D) high pressure and high temperature

5. Fig. shows graphs of pressure vs density for an ideal gas at two


temperatures T1 and T2.
(A) T1 > T2 (B) T1 = T2
(C) T1 < T2 (D) any of the three is possible
6. Suppose a container is evacuated to leave just one molecule of a gas in it. Let a and rms
represent the average speed and the rms speed of the gas.
(A) a > rms (B) a < rms (C) a = rms (D) rms is undefined
7. The rms speed of oxygen molecules in a gas is . If the temperature is doubled and the O2
molecule dissociate into oxygen atoms, the rms speed will become
(A)  (B) 2 (C) 2  (D) 4
8. Consider a mixture of oxygen and hydrogen kept at room temperature. As compared to a
hydrogen molecule an oxygen molecule hits the wall
(A) With greater average speed (B) with smaller average speed
(C) with greater average kinetic energy (D) with smaller average kinetic energy.

DOWNLOAD OUR APP & GET : 353, Rajeev Gandhi Nagar, Instrumentation Limited Colony, Kota, Rajasthan 324005

UNLIMITED PRACTICE FOR FREE : www.atpstar.com PAGE NO.-13


9. Consider the quantity MkT / pV of an ideal gas where M is the mass of the gas. It depends on
the
(A) temperature of the gas (B) volume of the gas
(C) pressure of the gas (D) nature of the gas
10. The ratio of the mean speed of hydrogen molecules to the mean speed of nitrogen molecules
in a sample containing a mixture of the two gases.
(A) 14 (B) 7 (C) 28 (D) None of these

11. A certain gas is taken to the five states represented by dots in the
graph. The plotted lines are isotherms. Order of the most probable
speed vp of the molecules at these five states is :
(A) VP at 3 > VP at 1 = VP at 2 > VP at 4 = VP at 5
(B) VP at 1 > VP at 2 = VP at 3 > VP at 4 > VP at 5
(C) VP at 3 > VP at 2 = VP at 4 > VP at 1 > VP at 5
(D) Insufficient information to predict the result.

DOWNLOAD OUR APP & GET : 353, Rajeev Gandhi Nagar, Instrumentation Limited Colony, Kota, Rajasthan 324005

UNLIMITED PRACTICE FOR FREE : www.atpstar.com PAGE NO.-14


PH YSI CS

DPP
DAILY PRACTICE PROBLEMS

TARGET : JEE(MAIN + ADVANCED)


ST

NO. 9
HEAT AND THERMODYNAMICS
TOPIC: THERMODYNAMICS

3PV
1. The pressure of an ideal gas is written as E = . Here E stands for
2
(A) average translational kinetic energy (B) rotational kinetic energy
(C) total kinetic energy. (D) None of these

2. The quantities which remain same for all ideal gases at the same temperature is/are?
(A) the kinetic energy of equal moles of gas
(B) the kinetic energy of equal mass of gas
(C) the number of molecules of equal moles of gas
(D) the number of molecules of equal mass of gas

3. Refer to fig. Let U1 and U2 be the changes in internal energy of an
ideal gas in the processes A and B then
(A) U1 > U2 (B) U1 = U2
(C) U1 < U2 (D) U1  U2

2U
4. The quantity represents (where U = internal energy of gas)
fkT
(A) mass of the gas (B) kinetic energy of the gas
(C) number of moles of the gas (D) number of molecules in the gas

5. In the following figures (A) to (D), variation of volume by change of pressure is shown. A gas
is taken along the path ABCDA. The change in internal energy of the gas will be:

(A) (B) (C) (D)

(A) positive in all cases from (A) to (D)


(B) positive in cases (A), (B) and (C) but zero in case (D)
(C) negative in cases (A), (B) and (C) but zero in case (D)
(D) zero in all the four cases.

DOWNLOAD OUR APP & GET : 353, Rajeev Gandhi Nagar, Instrumentation Limited Colony, Kota, Rajasthan 324005

UNLIMITED PRACTICE FOR FREE : www.atpstar.com PAGE NO.-15


6. An ideal gas changes from state a to state b as shown in Fig. What is
the work done by the gas in the process?
(A) zero (B) positive (C) negative (D) infinite

7. The process U = 0, for an ideal gas can be best represented in the form of a graph :
A
P

(A) (B) B (C) (D)


V

8. In the following V-T diagram what is the relation between P1 and P2 :


(A) P2 = P1
(B) P2 > P1
(C) P2 < P1
(D) cannot be predicted

9. In the isothermal expansion of an ideal gas. Select wrong statement:


(A) there is no change in the temperature of the gas
(B) there is no change in the internal energy of the gas
(C) the work done by the gas is equal to the heat supplied to the gas
(D) the work done by the gas is equal to the change in its internal energy

10. Consider two processes on a system as shown in figure. The


volumes in the initial states are the same in the two processes and
the volumes in the final states are also the same. Let W 1 and W 2
be the work done by the system in the processes A and B
respectively.
(A) W 1 > W 2
(B) W 1 = W 2
(C) W 1 < W 2
(D) Nothing can be said about the relation between W 1 and W 2

11. A mass of an ideal gas undergoes a reversible isothermal compression. Its molecules will
then have compared with initial state, the same
(i) root mean square velocity (ii) mean momentum
(iii) mean kinetic energy
(A) (i), (ii), (iii) correct (B) (i), (ii) correct
(C) (ii), (iii) correct (D) (i) correct

12. Ideal gas is taken through process shown in figure:


(A) In process AB, work done by system is positive
(B) In process AB, heat is rejected out of the system.
(C) In process AB, internal energy increases
(D) In process AB internal energy decreases and in process BC
internal energy increases.

DOWNLOAD OUR APP & GET : 353, Rajeev Gandhi Nagar, Instrumentation Limited Colony, Kota, Rajasthan 324005

UNLIMITED PRACTICE FOR FREE : www.atpstar.com PAGE NO.-16


PH YSI CS

DPP
DAILY PRACTICE PROBLEMS

TARGET : JEE(MAIN + ADVANCED)


ST

NO. 10
HEAT AND THERMODYNAMICS
TOPIC: THERMODYNAMICS

1. A system can be taken from the initial state p1, V1 to the final state p2, V2 by two different
methods. Let Q and W represent the heat given to the system and the work done by the
system. Which of the following must be the same in both the methods?
(A) Q (B) W (C) Q + W (D) Q - W

2. In isothermal process if heat is released from an ideal gas then,


(A) the internal energy of the gas will increase
(B) the gas will do positive work
(C) the gas will do negative work
(D) the given process is not possible

3. In an isothermal expansion of an ideal gas. Select wrong statement:


(A) there is no change in the temperature of the gas
(B) there is no change in the internal energy of the gas
(C) the work done by the gas is equal to the heat supplied to the gas
(D) the work done by the gas is equal to the change in its internal energy

4. When an ideal diatomic gas is heated at constant pressure , the fraction of the heat energy
supplied which increases the internal energy of the gas is .
2 3 3 5
(A) (B) (C) (D)
5 5 7 7

5. Boiling water is changing into steam. Under this condition, the specific heat of water is
(A) zero (B) one (C) Infinite (D) less than one

6. Supposing the distance between the atoms of a diatomic gas to be constant, its specific heat
at constant volume per mole (gram mole) is
5 3 7
(A) R (B) R (C) R (D) R
2 2 2

7. For an ideal gas, the heat capacity at constant pressure is larger than than that at constant
volume because
(A) positive work is done during expansion of the gas by the external pressure
(B) positive work is done during expansion by the gas against external pressure
(C) positive work is done during expansion by the gas against intermolecular forces of
attraction
(D) more collisions occur per unit time when volume is kept constant.
DOWNLOAD OUR APP & GET : 353, Rajeev Gandhi Nagar, Instrumentation Limited Colony, Kota, Rajasthan 324005

UNLIMITED PRACTICE FOR FREE : www.atpstar.com PAGE NO.-17


8. A gas has :
(A) one specific heat only (B) two specific heats only
(C) infinite number of specific heats (D) no specific heat

9. A gas is contained in a metallic cylinder fitted with a piston. The piston is suddenly moved in
to compress the gas and is maintained at this position. As time passes, after this pressure of
the gas in the cylinder
(A) increases
(B) decreases
(C) remains constant
(D) increases or decreases depending on the nature of the gas.

10. Two sample A and B are initially kept in the same state. The sample A is expanded through
an adiabatic process and the sample B through an isothermal process upto the same final
volume. The final pressures in A and B are pA and pB respectively.
(A) pA > pB
(B) pA = pB
(C) pA < pB
(D) The relation between pA and pB cannot be deduced

11. Let Ta and Tb be the final temperature of the samples A and B respectively in the previous
question then
(A) Ta < Tb
(B) Ta = Tb
(C) Ta > Tb
(D) The relation between Ta and Tb cannot be deduced.

12. Let W a and W b be the work done by the systems A and B respectively in the previous
question then :
(A) W a > W b
(B) W a = W b
(C) W a < W b
(D) The relation between W a and W b cannot be deduced

DOWNLOAD OUR APP & GET : 353, Rajeev Gandhi Nagar, Instrumentation Limited Colony, Kota, Rajasthan 324005

UNLIMITED PRACTICE FOR FREE : www.atpstar.com PAGE NO.-18


ANSWER KEY
DPP – 1
1. 64 J 2. 5 × 10–5 g/s 3. 4.0 W 4. 2:
t1  t 2  t 3
5. t1 t 2 t 3
6. (B) 7. (a) (A), (b) (D) 8. (B)
 
k1 k 2 k 3

DPP – 2
2 K (T2  T1 )
1. (C) 2. (C) 3. 15 W/m–°C 4.
In ( R2 / R1 )

5. (C) 6. 12 °C/s 7. (B) 8. (B)

DPP – 3
q1  q2
1. (D) 2. 3. 0.73 W. 4. 6 x 103 K; 4 x 103 K
q1

1
5. 6. 5803 7. 1.71 rc 8. (C)
90
DPP – 4
1. (B) 2. 7 minutes 3. (B) 4. (D) 5. (C)
6. (D) 7. (A) 8. (C)

DPP – 5
1. (B) 2. (A) 3. (B) 4. (A) 5. (B)
6. (A)

DPP – 6
1. (D) 2. (A) 3. (C) 4. (D) 5. (C)
6. (B) 7. (B) 8. (A) 9. (B)

DPP – 7
1. (A) 2. (C) 3. (C) 4. (B) 5. (B)
6. (B) 7. (A) 8. (A) 9. (A) 10. (D)
11. (D) 12. (D) 13. (D) 14. (C) 15. (B)

DPP – 8
1. (C) 2. (B) 3. (D) 4. (B) 5. (A)
6. (C) 7. (C) 8. (B) 9. (D) 10. (A)
11. (A)

DOWNLOAD OUR APP & GET : 353, Rajeev Gandhi Nagar, Instrumentation Limited Colony, Kota, Rajasthan 324005
UNLIMITED PRACTICE FOR FREE
: www.atpstar.com PAGE NO.-38
DPP – 9
1. (1) 2. (3) 3. (2) 4. (4) 5. (4)
6. (1) 7. (2) 8. (3) 9. (4) 10. (3)
11. (1) 12. (2)

DPP – 10
1. (4) 2. (3) 3. (4) 4. (4) 5. (3)
6. (1) 7. (2) 8. (3) 9. (2) 10. (3)
11. (1) 12. (3)

DOWNLOAD OUR APP & GET : 353, Rajeev Gandhi Nagar, Instrumentation Limited Colony, Kota, Rajasthan 324005
UNLIMITED PRACTICE FOR FREE
: www.atpstar.com PAGE NO.-39
PHYSICS

DPPDAILY PRACTICE PROBLEMS

TARGET : JEE(MAIN + ADVANCED)


ST NO. 1
PROPERTIES OF MATTER
TOPIC : ELASTICITY

1. If a compressive force of 3.0 × 104 N is exerted on the end of 20 cm long bone of cross-
sectional area 3.6cm2,
(a) will the bone break and
(b) if not, how much will it shorten?
Given, compressive strength of bone = 7.7 × 108 Nm–2 and Young's modulus of bone
= 1.5 × 1010Nm–2
2. Two exactly similar wires of steel and copper are stretched by equal force. If the difference in
their elongation is 0.5 cm, find by how much each wire has elongated. (Given Young's
modulus for steel = 2 × 1012 dyne cm–2 and for copper 12 × 1011 dyne cm–2).
3. Three blocks A, B and C each of mass 4 kg are attached as shown in figure. Both the wires
has equal cross sectional area 5 × 10–7 m2 . The surface is smooth. Find the longitudinal strain
in each wire if Young modulus of both the wires is 2 × 1011 N/m2 (take g = 10 m/s2 )

4. The diameter of a brass rod is 4 mm and Young’s modulus of brass is 9 × 1010 N/m2. The
force required to stretch it by 0.1% of its length is :
(A) 360 N (B) 36 N (C) 144 × 103 N (D) 36 × 105 N
5. A steel wire is suspended vertically from a rigid support. When loaded with a weight in air, it
expands by La and when the weight is immersed completely in water, the extension is reduced
to Lw. Then relative density of the material of the weight is
La Lw La Lw
(A) (B) (C) (D)
La  L w La Lw La  L w
6. Two wires of equal length and cross-section area suspended as shown in figure. Their
Young's modulus are Y1 and Y2 respectively. The equivalent Young's modulus will be

Y1  Y2 Y1Y2
(A) Y1 + Y2 (B) (C) Y  Y (D) Y1 Y2
2 1 2

DOWNLOAD OUR APP & GET : 353, Rajeev Gandhi Nagar, Instrumentation Limited Colony, Kota, Rajasthan 324005

UNLIMITED PRACTICE FOR FREE : www.atpstar.com PAGE NO.-43


7. The load versus elongation graph for four wires of the same materials is shown in the figure.
The thinnest wire is represented by the line :

(A) OC (B) OD (C) OA (D) OB


8. A force F is needed to break a copper wire having radius R. The force needed to break a
copper wire of radius 2 R will be :
(A) F/2 (B) 2 F (C) 4 F (D) F/4

DOWNLOAD OUR APP & GET : 353, Rajeev Gandhi Nagar, Instrumentation Limited Colony, Kota, Rajasthan 324005

UNLIMITED PRACTICE FOR FREE : www.atpstar.com PAGE NO.-44


PHYSICS

DPP
DAILY PRACTICE PROBLEMS

TARGET : JEE(MAIN + ADVANCED)


ST NO. 2
PROPERTIES OF MATTER
TOPIC : ELASTICITY

1. A bar of cross-section A is subjected to equal and opposite tensile forces F at its ends.
Consider a plane through the bar making an angle  with a plane at right angles to the bar

F  F

(a) What is the tensile stress at this plane in terms of F, A and ?
(b) What is the shearing stress at the plane, in terms of F, A and ?
(c) For what value of  is the tensile stress a maximum ?
(d) For what value of  is the shearing stress a maximum?
2. A square brass plate of side 1.0 m and thickness 0.005 m is subjected to a force F on its
smaller opposite edges, causing a displacement of 0.02 cm. If the shear modulus of brass is
0.4 × 1011 N/m2, the value of the force F is
(A) 4 × 103 N (B) 400 N (C) 4 × 104 N (D) 1000 N
3. Two hail stones with radii in the ratio of 1 : 2 fall from a great height through the atmosphere.
Then the ratio of their momentum after they have attained terminal velocity is
(A) 1 : 1 (B) 1 : 4 (C) 1 : 16 (D) 1 : 32
4. A 50 kg motor rests on four cylindrical rubber blocks. Each block has a height of 4 cm and a
cross-sectional area of 16 cm2. The shear modulus of rubber is 2 × 106 N/m2. A sideways
force of 500 N is applied to the motor. The distance that the motor moves sideways is
(A) 0.156 cm (B) 1.56 cm (C) 0.312 cm (D) 0.204 cm
5. A brass rod of length 2 m and cross-sectional area 2.0 cm2 is attached end to end to a steel
rod of length L and cross-sectional area 1.0 cm2 . The compound rod is subjected to equal and
opposite pulls of magnitude 5 × 104 N at its ends. If the elongations of the two rods are equal,
the length of the steel rod (L) is (YBrass = 1.0 × 1011 N/m2 and YSteel = 2.0 × 1011 N/m2 )
(A) 1.5 m (B) 1.8 m (C) 1 m (D) 2 m
6. A spherical ball is dropped in a long column of viscous liquid. Which of the following graphs
represent the variation of
F

P
Q

R
O t
(i) gravitational force with time
(ii) viscous force with time
(iii) net force acting on the ball with time
(A) Q, R, P (B) R, Q, P (C) P, Q, R (D) R, P, Q

DOWNLOAD OUR APP & GET : 353, Rajeev Gandhi Nagar, Instrumentation Limited Colony, Kota, Rajasthan 324005

UNLIMITED PRACTICE FOR FREE : www.atpstar.com PAGE NO.-45


7. The compressibility of water is 46.4 × 10–6/atm. This means that
(A) the bulk modulus of water is 46.4 × 106 atm
(B) volume of water decreases by 46.4 one-millionths of the original volume for each
atmosphere
increase in pressure
(C) when water is subjected to an additional pressure of one atmosphere, its volume
decreases by 46.4%
(D) When water is subjected to an additional pressure of one atmosphere, its volume is
reduced to 10–6 of its original volume.
8. If the ratio of lengths, radii and Young’s modulii of steel and brass wires in the figure are a, b
and c respectively. Then the corresponding ratio of increase in their lengths would be :
///////////////////////////////

Steel (LVhy )

Brass (ihry)

2m

2ac 3a 3c 2a2 c
(A) (B) (C) (D)
b2 2b2 c 2ab2 b

DOWNLOAD OUR APP & GET : 353, Rajeev Gandhi Nagar, Instrumentation Limited Colony, Kota, Rajasthan 324005

UNLIMITED PRACTICE FOR FREE : www.atpstar.com PAGE NO.-46


PHYSICS

DPPDAILY PRACTICE PROBLEMS

TARGET : JEE(MAIN + ADVANCED)


ST NO. 3
PROPERTIES OF MATTER
TOPIC : ELASTICITY

1. A spherical ball contracts in volume by 0.001% when it is subjected to a pressure of 100


atmosphere. Calculate its bulk modulus.

2. A metal block is experiencing an atmospheric pressure of 1 × 105 N/m2, when the same block
is placed in a vacuum chamber, the fractional change in its volume is (the bulk modulus of
metal is 1.25 × 1011 N/m2)
(A) 4 × 10–7 (B) 2 × 10–7 (C) 8 × 10–7 (D) 1 × 10–7

3. If a rubber ball is taken at the depth of 200 m in a pool its volume decreases by 0.1%. If the
density of the water is 1 × 103 kg/m3 and g = 10 m/s2, then the volume elasticity in N/m2 will be :
(A) 108 (B) 2 × 108 (C) 109 (D) 2 × 109

4. Two wires of the same material and length but diameter in the ratio 1 : 2 are stretched by the
same force. The ratio of potential energy per unit volume for the two wires when stretched will
be :
(A) 1 : 1 (B) 2 : 1 (C) 4 : 1 (D) 16 : 1

5. A small steel ball falls through a syrup at constant speed of 10 cm/s. If the steel ball is pulled
upwards with a force equal to twice its effective weight, how fast will it move upwards ?
(A) 10 cm/s (B) 20 cm/s (C) 5 cm/s (D) – 5 cm/s

6. Calculate the increase in energy of a brass bar of length 0.2 m and cross-sectional area 1 cm2
when compressed with a load of 5 kg-weight along its length.
(Young’s modulus of brass = 1.0 × 1011 N/m2 and g = 9.8 m/s2).

7. When the load on a wire increased slowly from 2 kg wt. to 4 kg wt., the elongation increases
from 0.6 mm to 1.00 mm. How much work is done during the extension of the wire? [g = 9.8
m/s2]

DOWNLOAD OUR APP & GET : 353, Rajeev Gandhi Nagar, Instrumentation Limited Colony, Kota, Rajasthan 324005

UNLIMITED PRACTICE FOR FREE : www.atpstar.com PAGE NO.-47


ANSWER KEY
DPP – 1
10
1. No, x 10–3 m = 1.11 mm.
9
2. 0.75 cm, 1.25 cm
4 8
3.  10 –4 ,  10 –4
3 3
4. (A) 5. (A) 6. (B) 7. (C) 8. (C)

DPP – 2
2
Fcos  F sin2
1. (a) (b) (c)  = 0º (d)  = 45º
A 2A
2. (C) 3. (D) 4. (A) 5. (D) 6. (C)
7. (B) 8. (B)

DPP – 3
7
1. 10 atmosphere 2. (C) 3. (D) 4. (D) 5. (A)
6. 2.4 × 10–5 J. 7. 13.72 × 10–3 J.

DOWNLOAD OUR APP & GET : 353, Rajeev Gandhi Nagar, Instrumentation Limited Colony, Kota, Rajasthan 324005
UNLIMITED PRACTICE FOR FREE
: www.atpstar.com PAGE NO.-52
PHYSICS

DPP
DAILY PRACTICE PROBLEMS

TARGET : JEE(MAIN + ADVANCED)


ST

NO. 1
FLUID MECHANICS
TOPIC : FLUID STATICS

1. We can cut an apple easily with a sharp knife as compared to a blunt knife. Explain why?
2. Why mercury is used in barometers instead of water?
3. Pressure 3 m below the free surface of a liquid is 15KN/m2 in excess of atmosphere pressure.
Determine its density and specific gravity. [g = 10 m/sec2]
4. Figure here shows the vertical cross-section of a vessel filled with a liquid of density . The
normal thrust per unit area on the walls of the vessel at point. P, as shown, will be

(A) h g (B) H g (C) (H – h) g (D) (H – h) g cos


5. A tank with length 10 m, breadth 8 m and depth 6m is filled with water to the top. If g = 10 m s–
2
and density of water is 1000 kg m–3, then the thrust on the bottom is (neglect atmospheric
pressure)
(A) 6 × 1000 × 10 × 80 N (B) 3 × 1000 × 10 × 48 N
(C) 3 × 1000 × 10 × 60 N (D) 3 × 1000 × 10 × 80 N
6. In a hydraulic lift, used at a service station the radius of the large and small piston are in the
ratio of 20 : 1. What weight placed on the small piston will be sufficient to lift a car of mass
1500 kg ?
(A) 3.75 kg (B) 37.5 kg (C) 7.5 kg (D) 75 kg.
7. Two vessels A and B of different shapes have the same base area and are filled with water up
to the same height h (see figure). The force exerted by water on the base is FA for vessel A
and FB for vessel B. The respective weights of the water filled in vessels are W A and W B. Then

(A) FA > FB ; W A > W B (B) FA = FB ; W A > W B


(C) FA = FB ; W A < W B (D) FA > FB ; W A = W B

DOWNLOAD OUR APP & GET : 353, Rajeev Gandhi Nagar, Instrumentation Limited Colony, Kota, Rajasthan 324005

UNLIMITED PRACTICE FOR FREE : www.atpstar.com PAGE NO.-54


8. (i) The cubical container ABCDEFGH which is completely filled with an ideal (nonviscous and
incompressible) fluid, moves in a gravity free space with a acceleration of
a = a0 (iˆ  ˆj  k)
ˆ

where a0 is a positive constant. Then the only point in the container shown in the figure where
pressure is maximum, is

(A) B (B) C (C) E (D) F


(ii) In previous question pressure will be minimum at point –
(A) A (B) B (C) H (D) F

DOWNLOAD OUR APP & GET : 353, Rajeev Gandhi Nagar, Instrumentation Limited Colony, Kota, Rajasthan 324005

UNLIMITED PRACTICE FOR FREE : www.atpstar.com PAGE NO.-55


PHYSICS

DPP
DAILY PRACTICE PROBLEMS

TARGET : JEE(MAIN + ADVANCED)


ST

NO. 2
FLUID MECHANICS
TOPIC : FLUID STATICS

1. A metallic square plate is suspended from a point x as shown in figure. The plate is made to
dip in water such that level of water is well above that of the plate. The point ‘x’ is then slowely
raised at constant velocity. Sketch the variation of tension T in string with the displacement ‘s’
of point x.

2. The density of ice is x gm/cc and that of water is y gm/cc. What is the change in volume in cc,
when m gm of ice melts?
(A) M (y – x) (B) (y – x)/m (C) mxy (x – y) (D) m (1/y – 1/x)
3. The reading of a spring balance when a block is suspended from it in air is 60 newton. This
reading is changed to 40 newton when the block is fully submerged in water. The specific
gravity of the block must be therefore :
(A) 3 (B) 2 (C) 6 (D) 3/2
4. A block of volume V and of density b is placed in liquid of density l(l > b), then block is
moved upward upto a height h and it is still in liquid. The increase in gravitational potential
energy of the block is :
(A) bVgh (B) (b + l)Vgh (C) (b – l)Vgh (D) none of these
5. A block of steel of size 5 cm × 5 cm × 5 cm is weighed in water. If the relative density of steel
is 7. Its apparent weight is :
(A) 6 × 5 × 5 × 5 gf (B) 4 × 4 × 4 × 7 gf (C) 5 × 5 × 5 × 7 gf (D) 4 × 4 × 4 × 6 gf
6. A metallic sphere floats in an immiscible mixture of water (w = 103 kg/m3) and a liquid
(L = 13.5 × 103) with (1/5)th portion by volume in the liquid and remaining in water. The
density of the metal is :
(A) 4.5 × 103 kg/m3 (B) 4.0 × 103 kg/m3 (C) 3.5 × 103 kg/m3 (D) 1.9 × 103 kg/m3
7. Two bodies are in equilibrium when suspended in water from the arms of a balance. The
mass of one body is 36 g and its density is 9 g/cc. If the mass of the other is 48 g, its density
in g/cc is :
(A) 4/3 (B) 3/2 (C) 3 (D) 5
8. In order that a floating object be in a stable equilibrium, its centre of buoyancy should be
(A) vertically above its centre of gravity
(B) vertically below its centre of gravity
(C) horizontally in line with its centre of gravity
(D) may be anywhere

DOWNLOAD OUR APP & GET : 353, Rajeev Gandhi Nagar, Instrumentation Limited Colony, Kota, Rajasthan 324005

UNLIMITED PRACTICE FOR FREE : www.atpstar.com PAGE NO.-56


PHYSICS

DPP
DAILY PRACTICE PROBLEMS

TARGET : JEE(MAIN + ADVANCED)


ST

NO. 3
FLUID MECHANICS
TOPIC : FLUID DYNAMICS

1. A fire hydrant (as shown in the figure) delivers water of density  at a volume rate L. The
water travels vertically upward through the hydrant and then does 900 turn to emerge
horizontally at speed V. The pipe and nozzle have uniform cross-section throughout. The
force exerted by the water on the corner of the hydrant is

(A) VL (B) zero (C) 2VL (D) 2VL


2. A tube in vertical plane is shown in figure. It is filled with a liquid of density and its end B is
closedThen the force exerted by the fluid on the tube at end B will be : [Neglect atmospheric
pressure and assume the radius of the tube to be negligible in comparison to ]

gA 0
(A) 0 (B) g A0 (C) 2g A0 (D)
2
3. A U-tube of base length “” filled with same volume of two liquids of densities  and 2 is
moving with an acceleration “a” on the horizontal plane as shown in the figure. If the height
difference between the two surfaces (open to atmosphere) becomes zero, then the height h is
given by:

a 3a a 2a
(A)  (B)  (C)  (D) 
2g 2g g 3g

DOWNLOAD OUR APP & GET : 353, Rajeev Gandhi Nagar, Instrumentation Limited Colony, Kota, Rajasthan 324005

UNLIMITED PRACTICE FOR FREE : www.atpstar.com PAGE NO.-57


4. A narrow tube completely filled with a liquid is lying on a series of cylinders as shown in figure.
Assuming no sliding between any surfaces, the value of acceleration of the cylinders for which
liquid will not come out of the tube from anywhere is given by
open to atmosphere
(ok;qe.My esa [kqyh gqbZ)
H
L

gH gH 2gH gH
(A) (B) (C) (D)
2L L L 2L
5. An open pan P filled with water (density w) is placed on a vertical rod, maintaining
equilibrium. A block of density  is placed on one side of the pan as shown in the figure.
Water depth is more than height of the block.
P

(A) Equilibrium will be maintained only if  < W .


(B) Equilibrium will be maintained only if   W .
(C) Equilibrium will be maintained for all relations between  and W .
(D) It is not possible to maintained the equilibrium
6. In the figure shown water is filled in a symmetrical container. Four pistons of equal area A are
used at the four opening to keep the water in equilibrium. Now an additional force F is applied
at each piston. The increase in the pressure at the centre of the container due to this addition
is

F 2F 4F
(A) (B) (C) (D) 0
A A A
7. A cube of wood supporting a 200 gm mass just floats in water. When the mass is removed the
cube rises by 2 cm at equilibrium. Find side of the cube.
8. A small solid ball of density half that of water falls freely under gravity from a height of 19.6 m
and then enters into water. Upto what depth will the ball go ? How much time will it take to
come again to the water surface? Neglect air resistance, viscosity effects of water and energy
loss due to collision at water surface.
(g = 9.8 m/s2)

DOWNLOAD OUR APP & GET : 353, Rajeev Gandhi Nagar, Instrumentation Limited Colony, Kota, Rajasthan 324005

UNLIMITED PRACTICE FOR FREE : www.atpstar.com PAGE NO.-58


PHYSICS

DPP
DAILY PRACTICE PROBLEMS

TARGET : JEE(MAIN + ADVANCED)


ST

NO. 4
FLUID MECHANICS
TOPIC : FLUID DYNAMICS

1. A cubical block of wood 10 cm on a side, floats at the interface of oil and water as shown in
figure. The density of oil is 0.6 g cm–3 and density of water is 1 g cm–3. The mass of the block
is

rsy dk ?kuRo

ikuh

(A) 706 g (B) 607 g (C) 760 g (D) 670 g


2. Figure shows a weighing-bridge, with a beaker P with water on one pan and a balancing
weight R on the other. A solid ball Q is hanging with a thread outside water. It has volume 40
cm3 and weighs 80 g. If this solid is lowered to sink fully in water, but not touching the beaker
anywhere, the balancing weight R' will be

(A) same as R (B) 40 g less than R


(C) 40 g more than R (D) 80 g more than R
3. A cylindrical container of radius ' R ' and height ' h ' is completely filled with a liquid. Two
horizontal L shaped pipes of small cross-section area ' a ' are connected to the cylinder as
shown in the figure. Now the two pipes are opened and fluid starts coming out of the pipes
horizontally in opposite directions. Then the torque due to ejected liquid on the system is:

(A) 4 a g h  R (B) 8 a g h  R (C) 2 a g h  R (D) aghR

DOWNLOAD OUR APP & GET : 353, Rajeev Gandhi Nagar, Instrumentation Limited Colony, Kota, Rajasthan 324005

UNLIMITED PRACTICE FOR FREE : www.atpstar.com PAGE NO.-59


4. A uniform rod OB of length 1m, cross-sectional area 0.012 m2 and relative density 2.0 is free
to rotate about O in vertical plane. The rod is held with a horizontal string AB which can
withstand a maximum tension of 45 N. The rod and string system is kept in water as shown in
figure. The maximum value of angle  which the rod can make with vertical without breaking
the string is

Fixed vessel
A B (tM+or~ ik=k)

a
O
(A) 45º (B) 37º (C) 53º (D) 60º
5. A non uniform cylinder of mass m, length  and radius r is having its centre of mass at a
distance /4 from the centre and lying on the axis of the cylinder as shown in the figure. The
cylinder is kept in a liquid of uniform density  The moment of inertia of the rod about the
centre of mass is . The angular acceleration of point A relative to point B just after the rod is
released from the position shown in figure is

(A) g r (B) g r (C) g r (D) 3g r


2 2 2 2 2 2 2 2

 4 2 4
6. A block of iron is kept at the bottom of a bucket full of water at 2°C. The water exerts buoyant
force on the block. If the temperature of water is increased by 1°C the temperature of iron
block also increases by 1°C. The buoyant force on the block by water
(A) will increase
(B) will decrease
(C) will not change
(D) may decrease or increase depending on the values of their coefficient of expansion
7. A liquid is kept in a cylindrical vessel which is rotated about its axis. The liquid rises at the
sides. If the radius of the vessel is 0.05 m and the speed of rotation is 2 rev/s, The difference
in the height of the liquid at the centre of the vessel and its sides will be (2 = 10) :
(A) 3 cm (B) 2 cm (C) 3/2 cm (D) 2/3 cm
8. A block is partially immersed in a liquid and the vessel is accelerating upwards with an
acceleration “a”. The block is observed by two observers O1 and O2 , one at rest and the other
accelerating with an acceleration “a” upward as shown in the figure. The total buoyant force
on the block is :

a a
O2

O1 (at rest)
(fLFkj)

(A) same for O1 and O2 (B) greater for O1 than O2


(C) greater for O2 than O1 (D) data is not sufficient

DOWNLOAD OUR APP & GET : 353, Rajeev Gandhi Nagar, Instrumentation Limited Colony, Kota, Rajasthan 324005

UNLIMITED PRACTICE FOR FREE : www.atpstar.com PAGE NO.-60


PHYSICS

DPP
DAILY PRACTICE PROBLEMS

TARGET : JEE(MAIN + ADVANCED)


ST

NO. 5
FLUID MECHANICS
TOPIC : FLUID DYNAMICS

1. Calculate the rate of flow of glycerin of density 1.25 x103 kg/m3 through the conical section of
a pipe placed horizontally, if the radii of its ends are 0.1m and 0.04 m and the pressure drop
across its length is 10 N/m².
2. Consider the Venturi tube of Figure. Let area A equal 5a. Suppose the pressure at A is 2.0
atm. Compute the values of velocity v at ‘A’ and velocity v at ‘a’ that would make the pressure
p at 'a' equal to zero. Compute the corresponding volume flow rate if the diameter at A is 5.0
cm. (The phenomenon at a when p falls to nearly zero is known as cavitation. The water
vaporizes into small bubbles.)
(Patm = 105 N/m2,  = 1000 kg/m3).

3. Water flows through a horizontal tube of variable cross-section (figure). The area of cross-
section at x and y are 40 mm2 and 20 mm2 respectively. If 10 cc of water enters per second
through x, find (i) the speed of water at x, (ii) the speed of water at y and (iii) the pressure
difference Px – Py .

4. Suppose the tube in the previous problem is kept vertical with x upward but the other
conditions remain the same. The separation between the cross-section at x and y is 15/16 cm.
Repeat parts (i), (ii) and (iii) of the previous problem. Take g = 10 m/s2.
5. Suppose the tube in the previous problem is kept vertical with y upward. Water enters through
y at the rate of 10 cm3/s. Repeat part (iii). Note that the speed decreases as the water falls
down.

DOWNLOAD OUR APP & GET : 353, Rajeev Gandhi Nagar, Instrumentation Limited Colony, Kota, Rajasthan 324005

UNLIMITED PRACTICE FOR FREE : www.atpstar.com PAGE NO.-61


6. Let air be at rest at the front edge of wing of an aeroplane and air passing over the surface of
the wing at a fast speed v. If density of air is , then find out the highest value for v in stream
line flow when atmospheric pressure is patm.
7. A tank is filled with water up to height H. Water is allowed to come out of a hole P in one of
the walls at a depth D below the surface of water as shown in the figure. Express the
horizontal distance x in terms of H and D :

(A) x = D(H  D) (B) x = D(H  D) (C) x = 2 D(H  D) (D) x = 4 D(H  D)


2

8. A fixed cylindrical vessel is filled with water up to height H. A hole is bored in the wall at a
depth h from the free surface of water. For maximum horizontal range h is equal to :
(A) H (B) 3H/4 (C) H/2 (D) H/4

DOWNLOAD OUR APP & GET : 353, Rajeev Gandhi Nagar, Instrumentation Limited Colony, Kota, Rajasthan 324005

UNLIMITED PRACTICE FOR FREE : www.atpstar.com PAGE NO.-62


PHYSICS

DPP
DAILY PRACTICE PROBLEMS

TARGET : JEE(MAIN + ADVANCED)


ST

NO. 6
FLUID MECHANICS
TOPIC : FLUID DYNAMICS

1. An incompressible liquid flows through a horizontal tube as shown in the figure. Then the
velocity ' v ' of the fluid is :

(A) 3.0 m/s (B) 1.5 m/s (C) 1.0 m/s (D) 2.25 m/s
2. For a fluid which is flowing steadily in a horizontal tube as shown in the figure, the level in the
vertical tubes is best represented by

(A) (B)

(C) (D)

3. There are two identical small holes on the opposite sides of a tank containing a liquid. The
tank is open at the top. The difference in height of the two holes is h as shown in the figure.
As the liquid comes out of the two holes, the tank will experience a net horizontal force
proportional to:

(A) h1/2 (B) h (C) h3/2 (D) h2


4. A cylindrical tank of height 0.4 m is open at the top and has a diameter 0.16 m. Water is filled
in it up to a height of 0.16 m. How long it will take to empty the tank through a hole of radius
5×10–3 m at its bottom ?
(A) 46.26 sec. (B) 4.6 sec. (C) 462.6 sec. (D) 0.46 sec.

DOWNLOAD OUR APP & GET : 353, Rajeev Gandhi Nagar, Instrumentation Limited Colony, Kota, Rajasthan 324005

UNLIMITED PRACTICE FOR FREE : www.atpstar.com PAGE NO.-63


5. A large cylindrical vessel contains water to a height of 10m. It is found that the thrust acting on
the curved surface is equal to that at the bottom. If atmospheric pressure can support a water
column of 10m, the radius of the vessel is

(A) 10 m (B) 15m (C) 5m (D) 25m

6. A jet of water of cross-sectional area A hits a plate normally with velocity v. the plate is moving
in the direction of the jet with velocity V. therefore, the force exerted on the plate is
proportional to

(A) v (B) v2 (C) (v-V) (D) (v-V)2


7. A portion of a tube is shown in the figure. Fluid is flowing from cross-section area A1 to A2. The
two cross-sections are at distance '  ' from each other. The velocity of the fluid at section A2 is

g
. If the pressures at A1 & A2 are same, then the angle made by the tube with the
2
horizontal will be:

3 3
(A) 37º (B) sin1 (C) 53º (D) cos1
4 4
8. There is a small hole in the bottom of a fixed container containing a liquid upto height ‘h’. The
top of the liquid as well as the hole at the bottom are exposed to atmosphere. As the liquid
comes out of the hole. (Area of the hole is ‘a’ and that of the top surface is ‘A’) :
(A) the top surface of the liquid accelerates with acceleration = g
a2
(B) the top surface of the liquid accelerates with acceleration = g
A2
a
(C) the top surface of the liquid retards with retardation = g
A
ga2
(D) the top surface of the liquid retards with retardation =
A2

DOWNLOAD OUR APP & GET : 353, Rajeev Gandhi Nagar, Instrumentation Limited Colony, Kota, Rajasthan 324005

UNLIMITED PRACTICE FOR FREE : www.atpstar.com PAGE NO.-64


PHYSICS

DPP
DAILY PRACTICE PROBLEMS

TARGET : JEE(MAIN + ADVANCED)


ST

NO. 7
FLUID MECHANICS
TOPIC : FLUID DYNAMICS

1. The velocity of the liquid coming out of a small hole of a large vessel containing two different
liquids of densities 2and as shown in figureis

(A) 6gh (B) 2 gh (C) 2 2gh (D) gh


2. Two water pipes P and Q having diameters 2 × 10–2 m and 4 × 10–2 m, respectively, are joined
in series with the main supply line of water. The velocity of water flowing in pipe P is
(A) 4 times that of Q (B) 2 times that of Q
(C) 1/2 times that of Q (D) 1/4 times that of Q
3. A large open tank has two holes in the wall. One is a square hole of side L at a depth y from
the top and the other is a circular hole of radius R at a depth 4y from the top. When the tank is
completely filled with water, the quantities of water flowing out per second from both holes are
the same. Then radius R, is equal to :

L L
(A) (B) 2 L (C) L (D)
2 2
4. A cup of water is placed in a car moving at a constant acceleration a to the left. Inside the
water is a small air bubble. The figure that correctly shows the shape of the water surface and
the direction of motion of the air bubble is.
a a a a

(A) (B) (C) (D)


(A) A (B) B (C) C (D) D

DOWNLOAD OUR APP & GET : 353, Rajeev Gandhi Nagar, Instrumentation Limited Colony, Kota, Rajasthan 324005

UNLIMITED PRACTICE FOR FREE : www.atpstar.com PAGE NO.-65


5. Two identical solid block A and B are made of two different materials. Block A floats in a liquid
with half of its volume submerged. When block B is pasted over A, the combination is found to
just float in the liquid. The ratio of the densities of the liquid, material of A and material of B is
given by
(A) 1 : 2 : 3 (B) 2 : 1 : 4 (C) 2 : 1 : 3 (D) 1 : 3 : 2
6. A hollow sphere of inner radius 9 cm and outer radius 10 cm floats half submerged in a liquid
of specific gravity 0.8. The density of the material of the spere is
(A) 0.84g cm–3 (B) 1. 48g cm–3 (C) 1. 84g cm–3 (D) 1.24g cm–3

DOWNLOAD OUR APP & GET : 353, Rajeev Gandhi Nagar, Instrumentation Limited Colony, Kota, Rajasthan 324005

UNLIMITED PRACTICE FOR FREE : www.atpstar.com PAGE NO.-66


PHYSICS

DPP
DAILY PRACTICE PROBLEMS

TARGET : JEE(MAIN + ADVANCED)


ST NO. 8
FLUID MECHANICS
TOPIC : SURFACE TENSION

1. A tube of 1 mm bore is dipped into a vessel containing a liquid of density 0.8 g/cm3, surface
tension 30 dyne/cm and angle of contact zero. Calculate the length which the liquid will
occupy in the tube when the tube is held (a) vertical (b) inclined to the vertical at an angle of
30º.
2. A soap film is stretched over a rectangular vertical wire frame as shown in the figure, what
forces hold section abcd in equilibrium ?

3. A mercury drop of radius 1.0 cm is sprayed into 106 droplets of equal size. Calculate the
energy expanded. (Surface tension of mercury = 32 × 10–2 N/m).
4. A film of water is formed between two straight parallel wires each 10 cm long and at
separation 0.5 cm. Calculate the work required to increase 1 mm distance between wires.
Surface tension = 7210 3 N/m.
5. A thread is tied slightly loose to a wire frame as shown in the figure. And the frame is dipped
into a soap solution and taken out. The frame is completely covered with the film. When the
portion A is punctured with a pin, the thread :
Frame (Ýse)
A

B Thread (/kkxk)

(A) becomes convex towards A


(B) becomes concave towards A
(C) remains in the initial position
(D) either (A) or (B) depending on size of A w.r.t. B
6. In a surface tension experiment with a capillary tube water rises upto 0.1 m. If the same
experiment is repeated in an artificial satellite, which is revolving around the earth ; water will
rise in the capillary tube upto a height of :
(A) 0.1 m (B) 0.2 m (C) 0.98 m (D) full length of tube
7. A thin metal disc of radius r floats on water surface and bends the surface downwards along
the perimeter making an angle  with vertical edge of the disc. If the disc displaces a weight of
water W and surface tension of water is T, then the weight of metal disc is :
(A) 2 rT + W (B) 2 rT cos – W (C) 2 rT cos + W (D) W – 2 rT cos
8. The surface tension of a liquid is 5 Newton per metre. If a film is held on a ring of area 0.02
metres2, its surface energy is about :
(A) 5 × 10–2 J (B) 2.5 × 10–2 J (C) 2 × 10–1 J (D) 3 × 10–1 J
DOWNLOAD OUR APP & GET : 353, Rajeev Gandhi Nagar, Instrumentation Limited Colony, Kota, Rajasthan 324005

UNLIMITED PRACTICE FOR FREE : www.atpstar.com PAGE NO.-67


PHYSICS

DPP
DAILY PRACTICE PROBLEMS

TARGET : JEE(MAIN + ADVANCED)


ST NO. 9
FLUID MECHANICS
TOPIC : SURFACE TENSION

1. The radii of the two columns is U-tube are r1 and r2. When a liquid of density  (angle of
contact is 0°) is filled in it, the level difference of liquid in two arms is h. The surface tension of
liquid is : (g = acceleration due to gravity) :
ghr1r2 gh(r2  r1 ) 2(r2  r1 ) gh
(A) (B) (C) (D)
2(r2  r1 ) 2r1r2 ghr1r2 2(r2  r1 )

2. Water rises in a capillary tube to a height h. it will rise to a height more than h
(A) on the surface of sun (B) in a lift moving down with an acceleration
(C) at the poles (D) in a lift moving up with an acceleration.
3. Insects are able to run on the surface of water because :
(A) insects have less weight
(B) insects swim on water
(C) of the Archimede’s upthrust
(D) surface tension makes the surface behave as elastic membrane.
4. A tube of fine bore AB is connected to a manometer M as shown. The stop cock S controls
the flow of air. AB is dipped into a liquid whose surface tension is . On opening the stop cock
for a while, a bubble is formed at B and the manometer level is recorded, showing a difference
h in the levels in the two arms. if  be the density of manometer liquid and r the radius of
curvature of the bubble, then the surface tension  of the liquid is given by
S
A

h M
B

rhg
(A) hrg (B) 2hgr (C) 4hrg (D)
4

DOWNLOAD OUR APP & GET : 353, Rajeev Gandhi Nagar, Instrumentation Limited Colony, Kota, Rajasthan 324005

UNLIMITED PRACTICE FOR FREE : www.atpstar.com PAGE NO.-68


5. Two parallel glass plates are dipped partly in the liquid of density ‘d’. keeping them vertical. If
the distance between the plates is ‘x’, Surface tension for liquid is T & angle of contact is 
then rise of liquid between the plates due to capillary will be :
T cos  2T cos  2T T cos 
(A) (B) (C) (D)
x d xdg xdgcos xdg
6. A soap bubble has radius R and surface tension S, How much energy is required to double
the radius without change of temperature.
7. The work done in blowing a bubble of volume V is W, then what is the work done in blowing a
soap bubble of volume 2V ?
8. Find the excess pressure inside a drop of mercury of radius 2 mm, a soap bubble of radius 4
mm and an air bubble of radius 4 mm formed inside a tank of water. Surface tension of
mercury is 0.465 N/m and soap solution and water are, 0.03 N/m and 0.076 N/m respectively.

DOWNLOAD OUR APP & GET : 353, Rajeev Gandhi Nagar, Instrumentation Limited Colony, Kota, Rajasthan 324005

UNLIMITED PRACTICE FOR FREE : www.atpstar.com PAGE NO.-69


PHYSICS

DPP
DAILY PRACTICE PROBLEMS

TARGET : JEE(MAIN + ADVANCED)


ST NO. 10
FLUID MECHANICS
TOPIC : SURFACE TENSION

1. Two identical soap bubbles each of radius r and of the same surface tension T combine to
form a new soap bubble of radius R. The two bubbles contain air at the same temperature. If
the atmospheric pressure is p0 then find the surface tension T of the soap solution in terms of
p0, r and R. Assume process is isothermal.
2. A spherical drop of water has 1mm radius. If the surface tension of the water is 50 × 10–3 N/m,
then find the difference of pressure between inside and outside the spherical drop is :
3. An empty container has a circular hole of radius r at its bottom. The container is pushed into
water very slowly as shown. To what depth the lower surface of container (from surface of
water) can be pushed into water such that water does not flow into the container ?

Water

(Surface tension of water = T, density of water = )


4. When charge is given to a soap bubble, it shows :
(A) a decrease in size (B) no change in size
(C) an increase in size (D) sometimes an increase and sometimes a decreases in size
5. A water drop is divided into 8 equal droplets. The pressure difference between the inner and
outer side of the big drop will be :
(A) same as for smaller droplet (B) 1/2 of that for smaller droplet
(C) 1/4 of that for smaller droplet (D) twice that for smaller droplet
6. An air bubble of radius r in water is at a depth h below the water surface at some instant. If P
is atmospheric pressure, d and T are density and surface tension of water respectively, the
pressure inside the bubble will be:
4T 2T 2T 4T
(A) P + h dg – (B) P + h dg + (C) P + h dg – (D) P + h dg +
r r r r
7. The work done to get n smaller equal size spherical drops from a bigger size spherical drop of
water is proportional to :
 1   1 
(A)  2 / 3   1 (B)  1/ 3   1 (C) n1/3 – 1 (D) n4/3 – 1
n  n 
8. Two unequal soap bubbles are formed one on each side of a tube closed in the middle by a
tap. What happens when the tap is opened to put the two bubbles in communication ?
(A) No air passes in any direction as the pressures are the same on two sides of the tap
(B) Larger bubble shrinks and smaller bubble increases in size till they become equal in size
(C) Smaller bubble gradually collapses and the bigger one increases in size
(D) None of the above

DOWNLOAD OUR APP & GET : 353, Rajeev Gandhi Nagar, Instrumentation Limited Colony, Kota, Rajasthan 324005

UNLIMITED PRACTICE FOR FREE : www.atpstar.com PAGE NO.-70


PHYSICS

DPP
DAILY PRACTICE PROBLEMS

TARGET : JEE(MAIN + ADVANCED)


ST NO. 11
FLUID MECHANICS
TOPIC : SURFACE TENSION

1. A soap bubble in vacuum has a radius of 3 cm and another soap bubble in vacuum has a
radius of 4 cm. If the two bubbles coalesce under isothermal conditions then the radius of the
new bubble is :
(A) 2.3 cm (B) 4.5 cm (C) 5 cm (D) 7 cm
2. A cylinder with a movable piston contains air under a pressure p1 and a soap bubble of radius
‘r’. The pressure p2 to which the air should be compressed by slowly pushing the piston into
the cylinder for the soap bubble to reduce its size by half will be : (The surface tension is ,
and the temperature T is maintained constant)
 24   24   24   12 
(A) 8p1  (B)  4p1  (C)  2p1  (D)  2p1 
 r   r   r   r 
3. A vessel whose bottom has round holes with a diameter of d = 0.1 mm is filled with water. The
maximum height of the water level h at which the water does not flow out, will be : (The water
does not wet the bottom of the vessel). [S.T of water = 70 dyn/cm]
(A) h = 24.0 cm (B) h = 25.0 cm (C) h = 26.0 cm (D) h = 28.0 cm
4. Water rises to a height h in a capillary tube lowered vertically into water to a depth  as shown
in the figure. The lower end of the tube is now closed, the tube is then taken out of the water
and opened again. The length of the water column remaining in the tube will be :

(A) 2h if > h and  + h if  < h (B) h if  > h and  + h if  < h


(C) 4h if  > h and  – h if  < h (D) h/2 if  > h and  + h if  < h
5. A soap bubble of radius r1 is placed on another soap bubble of radius r2(r1 < r2). The radius R
of the soapy film separating the two bubbles is :
r2r1
(A) r1 + r2 (B) r12  r22 (C) (r13 + r23) (D)
r2  r1
6. The high domes of ancient buildings have structural value (besides beauty). It arises from
pressure difference on the two faces due to curvature (as in soap bubbles). There is a dome
of radius 5 m and uniform (but small) thickness. The ‘surface tension’ of its masonary
structure is about 500 N/m. Treated as hemispherical, the maximum load the dome can
support is nearest to
(A) 1500 kg wt. (B) 3000 kg wt. (C) 6000 kg wt. (D) 12000 kg wt.
DOWNLOAD OUR APP & GET : 353, Rajeev Gandhi Nagar, Instrumentation Limited Colony, Kota, Rajasthan 324005

UNLIMITED PRACTICE FOR FREE : www.atpstar.com PAGE NO.-71


PHYSICS

DPP
DAILY PRACTICE PROBLEMS

TARGET : JEE(MAIN + ADVANCED)


ST NO. 12
FLUID MECHANICS
TOPIC : SURFACE TENSION

1. A soap - bubble with a radius ‘r’ is placed on another bubble with a radius R (figure). Angles
between the films at the points of contact will be –

(A) 1200 (B) 300 (C) 450 (D) 900

2. A large number of liquid drops each of radius 'a' coalesce to form a single spherical drop of
radius 'b'. The energy released in the process is converted into kinetic energy of the big drop
formed. The speed of big drop will be :

6T  1 1  4T  1 1  8T  1 1  5T  1 1 
(A)  (B)  (C)  (D) 
  a b    a b    a b    a b 

3. At critical temperature, the surface tension of a liquid :

(A) is zero

(B) is infinity

(C) is same as that at any other temperature

(D) cannot be determined

4. The excess pressure inside a soap bubble is equal to 2 mm of kerosene (density 0.8 g cm–3).
If the diameter of the bubble is 3.0 cm, the surface tension of soap solution is

(A) 39.2 dyne cm–1 (B) 45.0 dyne cm–1 (C) 51.1 dyne cm–1 (D) 58.8 dyne cm–1

DOWNLOAD OUR APP & GET : 353, Rajeev Gandhi Nagar, Instrumentation Limited Colony, Kota, Rajasthan 324005

UNLIMITED PRACTICE FOR FREE : www.atpstar.com PAGE NO.-72


PH YSI CS

DPP
DAILY PRACTICE PROBLEMS

TARGET : JEE(MAIN + ADVANCED)


ST

NO. 13
FLUID MECHANICS
TOPIC : SURFACE TENSION

1. The excess pressure inside an air bubble of radius r just below the surface of water is p1. The excess
pressure inside a drop of the same radius just outside the surface is p2. If T is surface tension, then
(A) p1 = 2p2 (B) p1 = p2 (C) p2 = 2p1 (D) p2 = 0, p1  0
2. On dipping a capillary of radius 'r' in water, water rises upto a height H and potential energy of water is u1. If

u1
a capillary of radius 2r is dipped in water, then the potential energy is u2. The ratio
u 2 is :

(A) 2 : 1 (B) 1 : 2 (C) 4 : 1 (D) 1 : 1


3. In a surface tension experiment with a capillary tube water rises up to 0.1 m. If the same experiment is
repeated on an artificial satellite which is revolving round the earth, water will rise in the capillary tube up to
a height of
(A) 0.1 m (B) 0.98 m
(C) 9.8 m (D) full length of capillary tube
4. A liquid drop of diameter D breaks into 27 tiny drops. The resultant change in energy is :
(A) 2TD2 (B) 4TD2 (C) TD2 (D) None of these
5. The additional force required to lift a flat circular disc of radius 5 cm from the surface of water with surface
tension 75 dynes/cm, will be-
(A) 750 dyne (B) 750 dyne (C) 30 dyne (D) 60 dyne
6. The correct curve between the height of depression h of liquid in a capillary tube and its radius is-

(A) (B) (C) (D)

7. Two thin wooden sticks are floating on the surface of water close to each other. A hot needle touches the
water between them. Then the sticks will-
(A) come closer (B) move apart (C) stay as before (D) move erratically
8. By blowing air in a soap bubble, the radius is increased from r to 3r. Then the percentage increase in the
surface energy of the bubble is-
(A) 90% (B) 80% (C) 800% (D) 900%
9. Mercury does not stick to glass or wood rod. It indicates that the cohesive force of mercury is-
(A) less than adhesive force (B) equal to adhesive force
(C) more than adhesive force (D) zero
10. Excess pressure inside a soap bubble is-

(A) 1/r (B) r (C)  r (D) Independent of r


DOWNLOAD OUR APP & GET : 353, Rajeev Gandhi Nagar, Instrumentation Limited Colony, Kota, Rajasthan 324005

UNLIMITED PRACTICE FOR FREE : www.atpstar.com PAGE NO. 73


PH YSI CS

DPP
DAILY PRACTICE PROBLEMS

TARGET : JEE(MAIN + ADVANCED)


ST

NO. 14
FLUID MECHANICS
TOPIC : VISCOSITY
1. An oil drop falls through air with a terminal velocity of 5 × 10 –4 m/s.
(i) the radius of the drop will be :
(A) 2.5 × 10–6 m (B) 2 × 10–6 m (C) 3 × 10–6 m (D) 4 × 10–6 m
2. The terminal velocity of a sphere moving through a viscous medium is :
(A) directly proportional to the radius of the sphere
(B) inversely proportional to the radius of the sphere
(C) directly proportional to the square of the radius of sphere
(D) inversely proportional to the square of the radius of sphere
3. A solid sphere falls with a terminal velocity of 10 m/s in air. If it is allowed to fall in vacuum,
(A) terminal velocity will be more than 10 m/s (B) terminal velocity will be less than 10 m/s
(C) terminal velocity will be 10 m/s (D) there will be no terminal velocity
4. A ball of mass m and radius r is released in a viscous liquid. The value of its terminal velocity is proportional to :
1 m m
(A) (B) (C) (D) m only
r r r
5. A spherical ball is dropped in a long column of viscous liquid. Which of the following graphs represent the
variation of F
(i) gravitational force with time P
(ii) viscous force with time Q

(iii) net force acting on the ball with time.


(A) Q, R, P (B) R, Q, P R
(C) P, Q, R (D) P, R, Q O t

6. In Poiseuilli's method of determination of coefficient of viscosity. the physical quantity that requires greater
accuracy in measurement is
(A) Pressure difference (B) Volume of the liquid collected
(C) Length of the capillary tube (D) Inner radius of the capillary tube
7. More viscous oil is used in summer than in winter in motors due to :–
(A) Rise in temperature in summer, the viscosity of oil decreases
(B) Rise in temperature in summer, viscosity of oil increases
(C) Surface tension of oil increases
(D) Surface tension of oil decreases
8. A rain drop of radius 0.3 mm has a terminal velocity in air 1m/s. The viscosity of air is 18 × 10–5 poise. The
viscous force on it is :–
(A) 101.73 × 10–4 dyne (B) 101.73 × 10–5 dyne
–5
(C) 16.95 × 10 dyne (D) 16.95 × 10–4 dyne
9. The velocity of falling rain drops attain, limited value because of :–
(A) Surface tension (B) Upthrust due to air
(C) Viscous force exerted by air (D) Air current
10. A copper ball of radius 'r' travels with a uniform speed 'v' in a viscous fluid. If the ball is changed with another
ball of radius '2r', then new uniform speed will be :–
(A) v (B) 2v (C) 4v (D) 8v
11. A spherical ball of radius 3.0 × 10–4 m and density 104 kg/m3 falls freely under gravity through a distance
h before entering a tank of water. If after entering the water the velocity of the ball does not change,
find h. Viscosity of water is 9.8 × 10–6 N-s/m2. [g = 9.8 m/s2]

DOWNLOAD OUR APP & GET : 353, Rajeev Gandhi Nagar, Instrumentation Limited Colony, Kota, Rajasthan 324005

UNLIMITED PRACTICE FOR FREE : www.atpstar.com PAGE NO. 74


ANSWER KEY
DPP – 1
3
3. 500 kg/m , 0.5 4. (C) 5. (A) 6. (A)
7. (B)
8. (i) (A)
(ii) (C)
DPP – 2

1.

2. (D) 3. (A) 4. (A) 5. (A) 6. (C)


7. (C) 8. (A)
DPP – 3
1. (D) 2. (B) 3. (B) 4. (A) 5. (B)
6. (A) 7. 10 cm 8. 19.6 m, 4 sec

DPP – 4
1. (C) 2. (C) 3. (A) 4. (B) 5. (B)
6. (A) 7. (B) 8. (A)
DPP – 5
4 3
1. 6.43 x 10 m /s
10 50
2. v= m/s = 4.1 m/s ; v = m/s = 21 m/s ;
6 6
Av = 8.1 x 103 m3 /sec
3. (i) 25 cm/s, (ii) 50 cm/s (iii) 93.75 N/m2
4. (i) 25 cm/s, (ii) 50 cm/s (iii) zero
1/ 2
2  2patm 
5. 187.5 N/m 6. vmax =   7. (C) 8. (C)
  
DPP – 6
1. (C) 2. (A) 3. (B) 4. (A) 5. (B)
6. (D) 7. (B) 8. (D)
DPP – 7
1. (B) 2. (A) 3. (A) 4. (D) 5. (C)
6. (B)

DPP – 8
1. (a) 1.53 cm, (b) 1.77 cm
2. Surface tension forces Fab, Fcd and weight. Equilibrium only when Fab > Fcd and this is due to
difference in concentration of soap solution in film.
3. 3.98 × 10–2 J
4. 1.44  10 5 J
5. (B) 6. (D) 7. (C) 8. (C)

DOWNLOAD OUR APP & GET : 353, Rajeev Gandhi Nagar, Instrumentation Limited Colony, Kota, Rajasthan 324005
UNLIMITED PRACTICE FOR FREE
: www.atpstar.com PAGE NO.-89
DPP – 9
1. (A) 2. (B) 3. (D) 4. (D) 5. (B)
6. 24 R2S 7. 22/3 W
8. (a) 465 N/m2 (b) 30 N/m2 (c) 38 N/m2

DPP – 10
3 3
p0 (2r  R ) 2T
1. T= 2. 100N/m2 3. 4. (C)
4(R 2  2r 2 ) gr
5. (B) 6. (B) 7. (C) 8. (C)

DPP – 11
1. (C) 2. (A) 3. (D) 4. (A) 5. (D)
6. (B)

DPP – 12
1. (A) 2. (A) 3. (A) 4. (D)

DPP – 13
1. (A) 2. (D) 3. (D) 4. (A) 5. (A)
6. (B) 7. (A) 8. (C) 9. (C) 10. (A)

DPP – 14
1. (C) 2. (C) 3. (D) 4. (B) 5. (C)
6. (D) 7. (A) 8. (A) 9. (C) 10. (C)
81
11. × 103 m
49

DOWNLOAD OUR APP & GET : 353, Rajeev Gandhi Nagar, Instrumentation Limited Colony, Kota, Rajasthan 324005
UNLIMITED PRACTICE FOR FREE
: www.atpstar.com PAGE NO.-90
PHYSICS

DPP
DAILY PRACTICE PROBLEMS

TARGET : JEE(MAIN + ADVANCED)


ST

NO. 1
WAVE ON A STRING
TOPIC: Basic Question

1. The property of a medium necessary for Mechanical wave propagation is -


(A) Its inertia (B) Its elasticity
(C) Its low resistance (D) All of above

2. Water wave are -


(A) Transverse
(B) Longitudinal
(C) Sometimes longitudinal some time transverse
(D) Neither transverse nor longitudinal

3. Superposition is the main characteristic of


(A) wave motion (B) particle motion
(C) wave and particle motion (D) none of the two motions

4. Which of the following are longitudinal waves ?


(A) sound waves (B) radio waves
(C) infrared waves (D) electromagnetic waves

5. Three consecutive flash photographs of a travelling wave


on a string are reproduced in the figure here. The
following observations are made. Mark the one which is
correct. (Mass per unit length of the string = 3 g/cm.)
(A) displacement amplitude of the wave is 0.25 m,
wavelength is 1 m, wave speed is 2.5 m/s and the
frequency of the driving force is 0.2/s.
(B) displacement amplitude of the wave is 2.0 m,
wavelength is 2 m, wave speed is 0.4 m/s and the
frequency of the driving force is 0.7/s.
(C) displacement amplitude of the wave is 0.25 m,
wavelength is 2 m, wave speed is 5 m/s and the
frequency of the driving force is 2.5 /s.
(D) displacement amplitude of the wave is 0.5 m,
wavelength is 2 m, wave speed is 2.5 m/s and the
frequency of the driving force is 0.2/s.

6. A certain transverse sinusoidal wave of wavelength 20 cm is


moving in the positive x direction. The transverse velocity of
the particle at x = 0 as a function of time is shown. The
amplitude of the motion is:
5 
(A) cm (B) cm
 2
10
(C) cm (D) 2 cm

: 353, Rajeev Gandhi Nagar, Instrumentation Limited Colony, Kota, Rajasthan 324005
DOWNLOAD OUR APP & GET
UNLIMITED PRACTICE FOR FREE : www.atpstar.com
PAGE NO.-93
7. Which of the following function correctly represents the progressive wave equation for finite values of
x and t
2 2 2
(A) y = x – t (B) y = cosx sint
2 2 2x
(C) y = log (x – t ) – log(x – t) (D) y = e sint

8. The equation of a wave is represented by :-


 x 
y = 10-4 sin  100t  m, where x and y are in meter and t in second; then the velocity of wave will
 10 
be:-
(A) 0.01 m/s. (B) 10 m/s. (C) 100 m/s . (D) 1000 m/s.

9. The phase difference between two waves, represented by


–6
y1 = 10 sin {100t + (x/50) + 0.5} m
–6  x 
y2 = 10 cos {100t +  }m
 50 
where x is expressed in meters and t is expressed in seconds, is approximately:-
(A) 1.07 rad (B) 2.07 rad (C) 0.5 rad (D) 1.5 rad

10. A wave in a string has an amplitude of 2 cm. The wave travels in the +ve direction of x axis with a
speed of 128 ms–1 and it is noted that 5 complete waves fit in 4m length of the string. The equation
describing the wave is
(A) y = (0.02)m sin (7.85 x + 100 5t) (B) y = (0.02)m sin (15.7 x – 2010t)
(C) y = (0.02)m sin (15.7 x + 2010t) (D) y = (0.02)m sin (7.85 x – 100 5t)

 x 
11. Equation of a progressive wave y = a sin  400 t – then its velocity will be :
 0.85 
(A) 34 m/s (B) 40 m/s (C) 340 m/s (D) 400 m/s

12. A plane progressive wave is given by y = 25 cos (2t – x) Then the amplitude and frequency are
respectively
(A) 25, 100 (B) 25, 1 (C) 25, 2 (D) 50 , 2

13. If the equation of motion of waves is y = 0.3 sin (314 t – 1.57x), then the velocity of wave will be :
(A) 400 m/s (B) 300 m/s (C) 200 m/s (D) 100 m/s

: 353, Rajeev Gandhi Nagar, Instrumentation Limited Colony, Kota, Rajasthan 324005
DOWNLOAD OUR APP & GET
UNLIMITED PRACTICE FOR FREE : www.atpstar.com
PAGE NO.-94
PHYSICS

DPP
DAILY PRACTICE PROBLEMS

TARGET : JEE(MAIN + ADVANCED)


ST

NO. 2
WAVE ON A STRING
TOPIC: Equation of Travelling Wave

1. For the wave shown in figure, the equation for the wave,
travelling along +x axis with velocity 350 ms–1 when its
position at t = 0 is as shown
314
(A) 0.05 sin ( x – 27475 t)
4
379
(B) 0.05 sin ( x – 27475 t)
5
314
(C) 1 sin ( x – 27475 t)
4
289
(D) 0.05 sin ( x + 27475 t)
5

2. A transverse wave is described by the equation Y = Y0 sin 2 (ft – x/). The maximum particle velocity
is equal to four times the wave velocity if
(A)  =  Y0/4 (B)  =  Y0/2 (C)  =  Y0 (D)  = 2 Y0

3. A travelling wave on a string is given by y = A sin [x + t + ]. The displacement and velocity of
6
oscillation of a point  = 0.56 /cm,  = 12/sec, A = 7.5 cm, x = 1 cm and t = 1s is
(A) 4.6 cm, 46.5 cm s–1 (B) 3.75 cm, 77.94 cm s–1
–1
(C) 1.76 cm, 7.5 cm s (D) 7.5 cm, 75 cm s–1
4. A transverse wave of amplitude 0.50m, wavelength 1m and frequency 2 Hz is propagating in a string in
the negative x-direction. The expression form of the wave is
(A) y(x, t) = 0.5 sin (2x – 4t) (B) y(x, t) = 0.5 cos (2x + 4t)
(C) y(x, t) = 0.5 sin (x – 2t) (D) y(x, t) = 0.5 cos (2x – 2t)

5. Two blocks each having a mass of 3.2 kg are connected by a wire CD and the
system is suspended from the ceiling by another wire AB (figure). The linear
mass density of the wire AB is 10 g/m and that of CD is 8 g/m. The speed of a
transverse wave pulse produced in AB and in CD are :
(A) 79 m/s and 63 m/s (B) 63 m/s and 79 m/s
(C) 63 m/s in both (D) 79 m/s in both

6. A wave pulse is generated in a string that lies along x-axis. At the


points A and B, as shown in figure, if RA and RB are ratio of wave
speed to the particle speed respectively then :
(A) RA > RB
(B) RB > RA
(C) RA = RB
(D) Information is not sufficient to decide.

: 353, Rajeev Gandhi Nagar, Instrumentation Limited Colony, Kota, Rajasthan 324005
DOWNLOAD OUR APP & GET
UNLIMITED PRACTICE FOR FREE : www.atpstar.com
PAGE NO.-95
7. Wave pulse on a string shown in figure is moving to the right
without changing shape. Consider two particles at positions x1 =
1.5 m and x2 = 2.5 m. Their transverse velocities at the moment
shown in figure are along directions :
(A) positive y–axis and positive y–axis respectively
(B) negative y–axis and positive y–axis respectively
(C) positive y–axis and negative y–axis respectively
(D) negative y–axis and negative y–axis respectively

8. Both the strings, show in figure, are made of same material and have same
cross-section. The pulleys are light. The wave speed of a transverse wave in
the string AB is v 1 and in CD it is v2. The v1/v2 is
1
(A) 1 (B) 2 (C) 2 (D)
2

: 353, Rajeev Gandhi Nagar, Instrumentation Limited Colony, Kota, Rajasthan 324005
DOWNLOAD OUR APP & GET
UNLIMITED PRACTICE FOR FREE : www.atpstar.com
PAGE NO.-96
PHYSICS

DPP
DAILY PRACTICE PROBLEMS

TARGET : JEE(MAIN + ADVANCED)


ST

NO. 3
WAVE ON A STRING
TOPIC: Interference, Reflection, Transmission, Power Transmitted along the String

1. When two waves of the same amplitude and frequency but having a phase difference of , travelling
with the same speed in the same direction (positive x), interfere, then
(A) their resultant amplitude will be twice that of a single wave but the frequency will be same
(B) their resultant amplitude and frequency will both be twice that of a single wave
(C) their resultant amplitude will depend on the phase angle while the frequency will be the same
(D) the frequency and amplitude of the resultant wave will depend upon the phase angle.

2. The equation of displacement of two waves are given as


y1 = 10 sin (3t + /3)
y2 = 5 (sin 3t + 3 cos 3t), then what is the ratio of their amplitude -
(A) 1 : 2 (B) 2 : 1 (C) 1 : 1 (D) None of these

3. Two waves of equal amplitude A, and equal frequency travels in the same direction in a medium. The
amplitude of the resultant wave is
(A) 0 (B) A (C) 2A (D) between 0 and 2A

4. The following figure depicts a wave travelling in a medium.


Which pair of particles are in phase.
(A) A and D (B) B and F
(C) C and E (D) B and G

5. The rate of transfer of energy in a wave depends


(A) directly on the square of the wave amplitude and square of the wave frequency
(B) directly on the square of the wave amplitude and square root of the wave frequency
(C) directly on the wave frequency and square of the wave amplitude
(D) directly on the wave amplitude and square of the wave frequency

6. A wave travels on a light string.The equation of the wave is Y = A sin (kx – t + 30°).It is reflected from
a heavy string tied to an end of the light string at x = 0.If 64% of the incident energy is reflected the
equation of the reflected wave is
(A)Y = 0.8 A sin (kx – t + 30° + 180°) (B)Y = 0.8 A sin (kx + t + 30° + 180°)
(C)Y = 0.8 A sin (kx + t – 30°) (D)Y = 0.8 A sin (kx + t + 30°)

7. Two sinusoidal waves with same wavelengths and amplitude travel in opposite directions along a string
with a speed 10 ms–1. If the minimum time interval between instants when the string is flat is 0.5s, the
wavelength of the waves is :
(A) 25 m (B) 20 m (C) 15 m (D) 10 m

: 353, Rajeev Gandhi Nagar, Instrumentation Limited Colony, Kota, Rajasthan 324005
DOWNLOAD OUR APP & GET
UNLIMITED PRACTICE FOR FREE : www.atpstar.com
PAGE NO.-97
PHYSICS

DPP
DAILY PRACTICE PROBLEMS

TARGET : JEE(MAIN + ADVANCED)


ST

NO. 4
WAVE ON A STRING
TOPIC: Standing Wave and Resonance

1. There are some points in a stationary waves which -


(A) Are never at rest (B) Are always in motion
(C) Are at rest twice in each cycle (D) Are at rest once in each cycle

2. The wave produced in the wire of a sonometer are -


(A) Transverse, progressive and polarized (B) Longitudinal
(C) Transverse, stationary and polarized (D) Transverse, stationary and un polarised
x
3. The equation of a stationary wave is Y = 10 sin cos 20t. The distance between two consecutive
4
nodes in meters is -
(A) 4 (B) 2 (C) 5 (D) 8

4. Stationary wave is represented by


y = A sin (100t) cos (0.01x)
where y and A are in mm, t in sec, and x in m. The velocity of the component wave -
(A) 1 m/s (B) 102m/s (C) 104 m/s (D) not derivable

5. A stretched sonometer wire resonates at a frequency of 350 Hz and at the next higher frequency of
420 Hz. The fundamental frequency of this wire is
(A) 350 Hz (B) 5 Hz (C) 70 Hz (D) 170 Hz

6. Two stretched wires A and B of the same lengths vibrate independently. If the radius, density and
tension of wire A are respectively twice those of wire B, then the fundamental frequency of vibration of
A relative to that of B is
(A) 1 : 1 (B) 1 : 2 (C) 1 : 4 (D) 1 : 8

7. If the tension and diameter of a sonometer wire of fundamental frequency n is doubled and density is
halved then its fundamental frequency will become -
n n
(A) (B) 2 n (C) n (D)
4 2

: 353, Rajeev Gandhi Nagar, Instrumentation Limited Colony, Kota, Rajasthan 324005
DOWNLOAD OUR APP & GET
UNLIMITED PRACTICE FOR FREE : www.atpstar.com
PAGE NO.-98
PHYSICS

DPP
DAILY PRACTICE PROBLEMS

TARGET : JEE(MAIN + ADVANCED)


ST

NO. 5
SOUND WAVES
TOPIC: Equation of sound wave, wavelength, frequency, pressure and displacement amplitude and Speed of Sound

1. When sound wave is refracted from air to water, which of the following will remain unchanged?
(A) wave number (B) wavelength (C) wave velocity (D) frequency

2. The frequency of a man’s voice is 300 Hz and its wavelength is 1 meter. If the wavelength of a child’s
voice is 1.5 m, then the frequency of the child’s voice is:
(A) 200 Hz (B) 150 Hz (C) 400 Hz (D) 350 Hz

3. The ratio of speed of sound in neon to that in water vapours at any temperature (when molecular weight
of neon is 2.02 × 10–2 kg mol–1 and for water vapours is 1.8 × 10–2 kg mol–1)
(A) 1.06 (B) 1.60 (C) 6.10 (D) 15.2

4. Under similar conditions of temperature and pressure, In which of the following gases the
velocity of sound will be largest.
(A) H2 (B) N2 (C) He (D) CO2

5. If vm is the velocity of sound in moist air and vd is the velocity of sound in dry air, then -
(A) vd > v m (B) vd = v m (C) vd  v m (D) v m > vd

: 353, Rajeev Gandhi Nagar, Instrumentation Limited Colony, Kota, Rajasthan 324005
DOWNLOAD OUR APP & GET
UNLIMITED PRACTICE FOR FREE : www.atpstar.com
PAGE NO.-99
PHYSICS

DPP
DAILY PRACTICE PROBLEMS

TARGET : JEE(MAIN + ADVANCED)


ST

NO. 6
SOUND WAVES
TOPIC: Intensity of sound, decibel scale and interference
1. A person is talking in a small room and the sound intensity level is 60 dB everywhere within the room. If
there are eight people talking simultaneously in the room, what is the sound intensity level ?
(A) 60 dB (B) 69 dB (C) 74 dB (D) 81 dB
2. When two waves with same frequency and constant phase difference interfere,
(A) there is a gain of energy
(B) there is a loss of energy
(C) the energy is redistributed and the distribution changes with time
(D) the energy is redistributed and the distribution remains constant in time
3. The terms pitch, quality and loudness of sound are associated with the following, respectively-
(A) intensity, frequency and waveform (B) Frequency, intensity and waveform
(C) Frequency, waveform and intensity (D) Waveform, frequency and intensity
4. For a wave displacement amplitude is 10–8 m, density of air 1.3 kg m–3, velocity in air 340 ms–1 and
frequency is 2000 Hz. The intensity of wave is
(A) 5.3 × 10–4 Wm–2 (B) 5.3 × 10–6 Wm–2 (C) 3.5 × 10–8 Wm–2 (D) 3.5 × 10–6 Wm–2
5. The sound intensity is 0.008 W/m2 at a distance of 10 m from an isotropic point source of sound. The
power of the source is :
(A) 2.5 watt (B) 0.8 watt (C) 8 watt (D) 10 watt
6. S1 and S2 are two coherent sources of radiations separated by distance 100.25 , where  is the wave
length of radiation. S1 leads S2 in phase by /2. A and B are two points on the line joining S1 and S2 as
shown in figure. The ratio of amplitudes of component waves from source S1 and S2 at A and B are in
 
ratio 1:2. The ratio of intensity at A to that of B  A  is
 B 

1
(A)  (B) (C) 0 (D) 9
9
7. Two waves of intensity I and 4I superpose on each other. Then in interference, maximum and
minimum intensity are respectively -
(A) 3I and 2I (B) 25 I and 9 I (C) 9 I and I (D) 5 I and 3 I
8. If the ratio of two sound intensities is 1 : 16, the ratio of their amplitudes of sound waves will be-
1 1 1 1
(A) (B) (C) (D)
2 4 8 16
9. Two waves of same frequency and of intensity 0 and 90 produces interference. If at a certain point the
resultant intensity is 70 then the minimum phase difference between the two sound waves will be -
(A) 90º (B) 100º (C) 120º (D) 110º
10. If the ratio of amplitudes of two waves at any point in the medium is 1 : 3, then the ratio of maximum
and minimum intensities due to their superposition will be
(A) 2 : 3 (B) 3 : 1 (C) 2 : 1 (D) 4 :1

: 353, Rajeev Gandhi Nagar, Instrumentation Limited Colony, Kota, Rajasthan 324005
DOWNLOAD OUR APP & GET
UNLIMITED PRACTICE FOR FREE : www.atpstar.com
PAGE NO.-100
PHYSICS

DPP
DAILY PRACTICE PROBLEMS

TARGET : JEE(MAIN + ADVANCED)


ST

NO. 7
SOUND WAVES
TOPIC: Reflection of Sound Equation of Stationary Waves and Organ Pipes and Resonance

1. When a sound wave is reflected from a wall, the phase difference between the reflected and incident
pressure wave is:
(A) 0 (B)  (C) /2 (D) /4

2. In stationary waves displacement, antinodes are the points where there is -


(A) Minimum displacement and minimum pressure change
(B) Minimum displacement and maximum pressure change
(C) Maximum displacement and maximum pressure change
(D) Maximum displacement and minimum pressure change

3. If 1, 2, 3 are the wavelengths of the waves giving resonance in the fundamental, first and second
overtone modes respectively in a open organ pipe, then the ratio of the wavelengths 1 : 2 : 3, is :
(A) 1 : 2 : 3 (B) 1 : 3 : 5 (C) 1 : 1/2 : 1/3 (D) 1 : 1/3 : 1/5

4. An open organ pipe of length L vibrates in its fundamental mode. The pressure variation is maximum
(A) at the two ends (B) at the middle of the pipe
(C) at distance L/4 inside the ends (D) at distance L/8 inside the ends

5. The fundamental frequency of a closed organ pipe is same as the first overtone frequency of an open
pipe. If the length of open pipe is 50 cm, the length of closed pipe is
(A) 25 cm (B) 12.5 cm (C) 100 cm (D) 200 cm

6. A cylindrical tube, open at both ends, has a fundamental frequency . The tube is dipped vertically in
water so that half of its length is inside the water. The new fundamental frequency is
(A) /4 (B) /2 (C)  (D) 2

7. An open pipe of length 33 cm resonates to a frequency of 1000 Hz. The mode of vibration is: (velocity
of sound = 330 m/s)
(A) Fundamental (B) The 2nd harmonic (C) The 3rd harmonic (D) The 4th harmonic

8. In Resonance tube experiment, if 400 Hz tuning fork is used, the first resonance occurs when length of
air column in the tube is 19 cm. If the 400 Hz. tuning fork is replaced by 1600 Hz tuning fork then to get
resonance, the water level in the tube should be further lowered by (take end correction = 1 cm)
(A) 5 cm (B) 10 cm (C) 15 cm (D) 20 cm

: 353, Rajeev Gandhi Nagar, Instrumentation Limited Colony, Kota, Rajasthan 324005
DOWNLOAD OUR APP & GET
UNLIMITED PRACTICE FOR FREE : www.atpstar.com
PAGE NO.-101
PHYSICS

DPP
DAILY PRACTICE PROBLEMS

TARGET : JEE(MAIN + ADVANCED)


ST

NO. 8
SOUND WAVES
TOPIC: Beats and Doppler Effect

1. If two tuning forks A & B give 4 beats/sec. with each other, on loading A with wax, 2 beats/sec. are
given. If frequency of A is 256 Hz, then frequency of B is -

(A) 250 Hz (B) 252 Hz (C) 260 Hz (D) 262 Hz

2. The frequency of a fork A is 3% more than the frequency of a standard fork whereas the frequency of
fork B is 3% less. The forks A and B produce 6 beats per second. The frequency of stadard fork will be

(A) 100 Hz (B) 106 Hz (C) 103 Hz (D) 112 Hz

3. A closed organ pipe and an open pipe of same length produce 4 beats when they are set into vibrations
simultaneously. If the length of each of them were twice their initial lengths, the number of beats
produced will be [Assume same mode of vibration in both cases]

(A) 2 (B) 4 (C) 1 (D) 8

4. Two identical wires are stretched by the same tension of 101 N & each emits a note of frequency
202 Hz. If the tension in one wire is increased by 1 N, then the beat frequency is:

(A) 2 Hz (B) 1 Hz (C) 1 Hz (D) none of these


2

5. Each of the two string of length 51.6 cm and 49.1 cm are tensioned separately by 20 N force. Mass per
–1
unit length of both the strings is same and equal to 1 gm . When both the strings vibrate
simultaneously the number of beats is

(A) 5 (B) 7 (C) 8 (D) 3

6. Two identical straight wires are stretched so as to produce 6 beats per second when vibrating
simultaneously. On changing the tension slighthly in one of them, the beat frequency still remains
unchanged. Denoting by T1 and T2, the higher and the lower initial tensions in the strings, it could be
said that while making the above changes in tension :

(A) T1 was decreased (B) T1 was increased (C) T2 was decreased (D) None of these

7. The change in frequency due to Doppler effect does not depend on

(A) the speed of the source (B) the speed of the observer

(C) the frequency of the source (D) separation between the source and the observer

: 353, Rajeev Gandhi Nagar, Instrumentation Limited Colony, Kota, Rajasthan 324005
DOWNLOAD OUR APP & GET
UNLIMITED PRACTICE FOR FREE : www.atpstar.com
PAGE NO.-102
8. An engine driver moving towards a wall with velocity of 50 ms–1 emits a note
of frequency 1.2 kHz. The frequency of note after reflection from the wall as
–1
heard by the engine driver when speed of sound in air is 350 ms is :

(A) 1 kHz (B) 1.8 kHz

(C) 1.6 kHz (D) 1.2 kHz

9. Two trains move towards each other with the same speed. Speed
of sound is 340 ms–1. If the pitch of the tone of the whistle of one
when heard on the other changes by 9/8 times, then the speed of
each train is :
–1 –1
(A) 2 ms (B) 40 ms
–1 –1
(C) 20 ms (D) 100 ms

10. A receiver & a source of sonic oscillations of frequency 200 Hz are located on the x  axis. The receiver
is fixed and the source swings harmonically along that axis with a circular frequency  and an
amplitude 50 cm. At what value of  (in rad/sec) will the frequency band width (f max  f min) registered by
the stationary receiver be equal to 20 Hz. [ The velocity of sound is equal to 340 m/s ]

(A) 17 (B) 34 (C) 68 (D) 8.5

: 353, Rajeev Gandhi Nagar, Instrumentation Limited Colony, Kota, Rajasthan 324005
DOWNLOAD OUR APP & GET
UNLIMITED PRACTICE FOR FREE : www.atpstar.com
PAGE NO.-103
ANSWER KEY

DPP – 01
1. (4) 2. (3) 3. (1) 4. (1) 5. (3) 6. (3) 7. (3)
8. (4) 9. (1) 10. (4) 11. (3) 12. (2) 13. (3)

DPP - 02
1. (1) 2. (2) 3. (2) 4. (2) 5. (1) 6. (2) 7. (2)
8. (4)

DPP - 03
1. (3) 2. (3) 3. (4) 4. (4) 5. (1) 6. (3) 7. (4)

DPP - 04
1. (3) 2. (3) 3. (1) 4. (3) 5. (3) 6. (2) 7. (3)

DPP – 05
1. (4) 2. (1) 3. (1) 4. (1) 5. (4)

DPP – 06
1. (2) 2. (4) 3. (3) 4. (4) 5. (4) 6. (2) 7. (3)
8. (2) 9. (3) 10. (4)

DPP – 07
1. (1) 2. (4) 3. (3) 4. (2) 5. (2) 6. (3) 7. (2)
8. (1,3)

DPP – 08
1. (2) 2. (1) 3. (1) 4. (3) 5. (2) 6. (1) 7. (4)
8. (3) 9. (3) 10. (2)

: 353, Rajeev Gandhi Nagar, Instrumentation Limited Colony, Kota, Rajasthan 324005
DOWNLOAD OUR APP & GET
UNLIMITED PRACTICE FOR FREE : www.atpstar.com
PAGE NO.-121
PHYSICS

DPP
DAILY PRACTICE PROBLEMS

TARGET : JEE(MAIN + ADVANCED)


ST

NO. 1
MODERN PHYSICS
TOPIC : MP

1. When a light of wavelength 400 nm falls on a metal of work function 2.5 eV, what will be the
maximum magnitude of linear momentum of emitted photoelectron?
2. The electric field associated with a monochromatic light is given by E = E0 sin (1.2 × 1015 t -
kx). Find the maximum kinetic energy of the photoelectrons when this light falls on a metal
surface whose work function is 2.0 eV
3. One milliwatt of light of wavelength  = 4560 Å is incident on a cesium metal surface.
Calculate the electron current liberated. Assume a quantum efficiency of  = 0.5 %. [Work
function for cesium = 1.89 eV] Take hc = 12400 eV-Å.
4. Suppose the wavelength of the incident light in photoelectric effect experiment is increased
from 3000 Aº to 3040 Aº. Find the corresponding change in the stopping potential. [Take the
product hc = 12.4  107 eV m]
5. The magnetic field at a point associated with a light wave is B = 2 × 10–6 Tesla sin [(3.0 × 1015
s–1 )t] sin [(6.0 × 10 15 s – 1)t]. If this light falls on a metal surface having a work function of 2.0
eV, what will be the maximum kinetic energy of the photoelectrons ?
6. In an experiment on photoelectric effect, light of wavelength 800 nm (less than threshold
wavelength) is incident on a cesium plate at the rate of 5.0 W .The potential of the collector
plate is made sufficiently positive with respect to the emitter so that the current reaches its
saturation value .Assuming that on the average one of every 106 photons is able to eject a
photoelectron, find the photo current in the circuit.
7. In a photoelectric effect experiment, photons of energy 5 eV are incident on the photocathode
of work function 3 eV. For photon intensity A = 1015 m–2 s–1, saturation current of 4.0  A is
obtained. Sketch the variation of photocurrent ip against the anode voltage Va in the figure
below for photon intensity A (curve A) and B = 2 × 1015 m–2 s–1 (curve B) (in JEE graph was to
be drawn in the answer sheet itself.)

8
ip(A)
6

–6 –4 –2 0 2 4 6
VA (volts)
8. In a photoelectric experiment, if stopping potential is applied, then photocurrent becomes
zero. This means that :
(A) the emission of photoelectrons is stopped
(B) the photoelectrons are emitted but are reabsorbed by the emitter metal
(C) the photoelectrons are accumulated near the collector plate
(D) the photoelectrons are dispersed from the sides of the apparatus.

DOWNLOAD OUR APP & GET : 353, Rajeev Gandhi Nagar, Instrumentation Limited Colony, Kota, Rajasthan 324005

UNLIMITED PRACTICE FOR FREE : www.atpstar.com PAGE NO.-124


PHYSICS

DPP
DAILY PRACTICE PROBLEMS

TARGET : JEE(MAIN + ADVANCED)


ST

NO. 2
MODERN PHYSICS
TOPIC : MP

1. If the frequency of light in a photoelectric experiment is doubled then maximum kinetic energy
of photoelectron
(A) be doubled (B) be halved
(C) become more than double (D) become less than double
2. Two separate monochromatic light beams A and B of the same intensity (energy per unit area
per unit time) are falling normally on a unit area of a metallic surface. Their wavelength are A
and B respectively. Assuming that all the incident light is used in ejecting the photoelectrons,
the ratio of the number of photoelectrons from beam A to that from B is
2 2
 A   B   A   B 
(A)   (B)   (C)   (D)  
 B   A   B   A 
3. Which one of the following graphs in figure shows the variation of photoelectric current (I) with
voltage (V) between the electrodes in a photoelectric cell ?

(A) (B) (C) (D)

4. When a centimetre thick surface is illuminated with light of wavelength , the stopping
potential is V. When the same surface is illuminated by light of wavelength 2, the stopping
potential is V/3. The threshold wavelength for the surface is :
4 8
(A) (B) 4  (C) 6  (D)
3 3
5. The anode plate in an experiment on photoelectric effect is kept vertically above the cathode
plate. Light source is put on and a saturation photocurrent is recorded. An electric field is
switched on which has vertically downward direction
(A) The photocurrent will increase
(B) The kinetic energy of the electrons will increase
(C) The stopping potential will decrease
(D) The threshold wavelength will increase
6. The maximum kinetic energy of photoelectrons emitted from a surface when photons of
energy 6 eV fall on it is 4 eV. The stopping potential is :
(A) 2V (B) 4V (C) 6V (D) 10V
7. Ultraviolet light of wavelength 300 nm and intensity 1 W/m2 falls on the surface of a
photosensitive material. If one percent of the incident photons produce photoelectrons then
the number of photoelectrons emitted per second from an area of 1 cm2 of the surface is
nearly
(A) 1.51 × 1013 (B) 1.51 × 1012 (C) 4.12 × 1013 (D) 2.13 × 1011
8. The photoelectrons emitted from a metal surface :
(A) Are all at rest (B) Have the same kinetic energy
(C) Have the same momentum
(D) Have speeds varying from zero up to a certain maximum value

DOWNLOAD OUR APP & GET : 353, Rajeev Gandhi Nagar, Instrumentation Limited Colony, Kota, Rajasthan 324005

UNLIMITED PRACTICE FOR FREE : www.atpstar.com PAGE NO.-125


PHYSICS

DPP
DAILY PRACTICE PROBLEMS

TARGET : JEE(MAIN + ADVANCED)


ST

NO. 3
MODERN PHYSICS
TOPIC : MP
3 2
1. Intensity of sunlight falling normally on the earth surface is 1.4 × 10 W/m . Assume that the
light is monochromatic with average wavelength 5000Å and that no light is absorbed in
between the sun and the earth’s surface. The distance between the sun and the earth is
11
1.5 × 10 m.
(a) Calculate the number of the photons falling per second on each square meter of earth’s
surface directly below the sun.
(b) How many photons are there in each cubic meter near the earth’s surface at any instant ?
(c) How many photons does the sun emits per second ?
2. A parallel beam of monochromatic light of wavelength 663 nm is incident on a totally reflecting
plane mirror. The angle of incidence is 60° and the number of photons striking the mirror per
second is 5 × 1019. Calculate the force exerted by the light beam on the mirror.
(h = 6.63 × 10–34 J.s.)
3. A beam of white light is incident normally on a plane surface absorbing 70% of the light and
reflecting the rest. If the incident beam carries 30 W of power, find the force exerted by it on
the surface.
4. A sodium lamp of power 10 W is emitting photons of wavelength 590 nm. Assuming that 60%
of the consumed energy is converted into light, find the number of photons emitted per second
by the lamp.
5. A photon of light enters a block of glass after travelling through vacuum. The energy of the
photon on entering the glass block
(A) increases because its associated wavelength decreases
(B) Decreases because the speed of the radiation decreases
(C) Stays the same because the speed of the radiation and the associated wavelength do not
change
(D) Stays the same because the frequency of the radiation does not change
6. Photoelectrons are liberated by ultraviolet light of wavelength 3000 Å from a metallic surface
for which the photoelectric threshold wavelength is 4000 Å. Calculate the de Broglie
wavelength of electrons emitted with maximum kinetic energy.
7. Two identical nonrelativistic particles move at right angles to each other, possessing de-
Broglie wavelengths, 1 & 2. Find the de-Broglie wavelength of each particle in the frame of
their centre of mass.
8. The energy of a photon of frequency  is E = h and the momentum of a photon of
wavelength  is p = h/. From this statement one may conclude that the wave velocity of light
is equal to :
2
8 –1 E E
(A) 3 × 10 ms (B) (C) E p (D)  
p p

DOWNLOAD OUR APP & GET : 353, Rajeev Gandhi Nagar, Instrumentation Limited Colony, Kota, Rajasthan 324005

UNLIMITED PRACTICE FOR FREE : www.atpstar.com PAGE NO.-126


PHYSICS

DPP
DAILY PRACTICE PROBLEMS

TARGET : JEE(MAIN + ADVANCED)


ST

NO. 4
MODERN PHYSICS
TOPIC : MP
8 –1
1. The de Broglie wavelength of an electron moving with a velocity 1.5 × 10 ms is equal to that
of a photon. The ratio of the kinetic energy of the electron to that of the energy of photon is
1 1
(A) 2 (B) 4 (C) (D)
2 4
2. A particle of mass M at rest decays into two particles of masses m1 and m2 having non zero
velocities. The ratio of the de Broglie wavelengths of the particles, 1/2 is :
m1 m2 m2
(A) (B) (C) 1 : 1 (D)
m2 m1 m1

3. Let p and E denote the linear momentum and the energy of a photon. For another photon of
smaller wavelength (in same medium)
(A) both p and E increase (B) p increases and E decreases
(C) p decreases and E increases (D) both p and E decreases
4. The de Broglie wavelength of a neutron correspoding to root mean square speed at 927ºC is
. What will be the de Broglie wavelength of the neutron correspoding to root mean square
speed at 27ºC?

(A) (B)  (C) 2  (D) 4 
2
5. The wavelength  of de Broglie waves associated with an electron (mass m, charge e)
accelerated through a potential difference of V is given by (h is Planck’s constant) :
(A)  = h/mV (B)  = h/2 meV (C)  = h/ meV (D)  = h/ 2meV
6. Find the numerical value of de-Broglie wavelength of an electron in the 1st orbit of hydrogen
atom assuming Bohr’s atomic model. You can use standard values of the constants. Leave
your answer in terms of .
7. Find the radius and energy of a He+ ion in the states (a) n = 2, (b) n = 3.
8. A positive hydrogen like ion having electron at its ground state ejects it, if a photon of
wavelength 228 Å or less is absorbed by it. Identify the ion.
9. Find the temperature at which the average kinetic energy of the molecules of hydrogen equals
the binding energy of its electron in ground state, assuming average kinetic energy of
3
hydrogen gas molecule = kT .
2

DOWNLOAD OUR APP & GET : 353, Rajeev Gandhi Nagar, Instrumentation Limited Colony, Kota, Rajasthan 324005

UNLIMITED PRACTICE FOR FREE : www.atpstar.com PAGE NO.-127


PHYSICS

DPP
DAILY PRACTICE PROBLEMS

TARGET : JEE(MAIN + ADVANCED)


ST

NO. 5
MODERN PHYSICS
TOPIC : MP

1. A monochromatic light source of frequency  illuminates a metallic surface and ejects


photoelectrons. The photoelectrons having maximum energy are just able to ionize the
hydrogen atoms in ground state. When the whole experiment is repeated with incident
5
radiations of frequency    the photoelectrons so emitted are able to excite the hydrogen
6

atom which then emits a radiation of wavelength of 1215 Å. Find the frequency .
2. If a0 is the Bohr radius, the radius of the n = 2 electronic orbit in triply ionized beryllium is -
(A) 4a0 (B) a0 (C) a0/4 (D) a0/16
3. Consider 2 hydrogen like ions A and B. Ionization energy of A is greater than that of B. Let r,
u, E and L represent the radius of the orbit, speed of the electron, energy of the atom and
orbital angular momentum of the electron respectively. In ground state:
(A) rA > rB (B) uA > uB (C) EA > EB (D) LA > LB
4. In Bohr’s model of hydrogen atom, the centripetal force is provided by the Coulomb attraction
between the proton and the electron. If a0 is the radius of the ground state orbit, m is the mass

and e the charge of an electron and 0 is the vacuum permittivity, the speed of the electron is :

e e 40 a0m
(A) zero (B) (C) (D)
0 a0 m 40 a0m e

5. If an orbital electron of the hydrogen atom jumps from the ground state to a higher energy
state, its orbital speed reduces to half its initial value. If the radius of the electron orbit in the
ground state is r, then the radius of the new orbit would be :
(A) 2r (B) 4r (C) 8r (D) 16r
6. In the Bohr model of the hydrogen atom, the ratio of the kinetic energy to the total energy of
the electron in a quantum state n is :
1 1
(A) – 1 (B) + 1 (C) (D)
n n2
7. The innermost orbit of the hydrogen atom has a diameter of 1.06 Å. What is the diameter of
the tenth orbit ?
(A) 5.3 Å (B) 10.6 Å (C) 53 Å (D) 106 Å

DOWNLOAD OUR APP & GET : 353, Rajeev Gandhi Nagar, Instrumentation Limited Colony, Kota, Rajasthan 324005

UNLIMITED PRACTICE FOR FREE : www.atpstar.com PAGE NO.-128


PHYSICS

DPP
DAILY PRACTICE PROBLEMS

TARGET : JEE(MAIN + ADVANCED)


ST

NO. 6
MODERN PHYSICS
TOPIC : MP

1. The orbital speed of the electron in the ground state of hydrogen is v. What will be its orbital
speed when it is excited to the energy state – 3.4 eV ?
v v v
(A) 2 v (B) (C) (D)
2 4 8
2. The total energy of the electron in the first excited state of hydrogen is – 3.4 eV. What is the
kinetic energy of the electron in this state ?
(A) + 1.7 eV (B) + 3.4 eV (C) + 6.8 eV (D) – 13.4 eV
3. In above Q., the potential energy of the electron is :
(A) – 1.7 eV (B) – 3.4 eV (C) – 6.8 eV (D) – 13.4 eV
4. Imagine an atom made of a proton and a hypothetical particle of double the mass as that of
an electron but the same charge. Apply Bohr theory to consider transitions of the hypothetical
particle to the ground state. Then, the longest wavelength (in terms of Rydberge constant for
hydrogen atom) is
1 5 1 2
(A) (B) (C) (D)
2R 3R 3R 3R
5. The force of attraction between the positively charged nucleus and the electron in a hydrogen
e2
atom is given by f = k . Assume that the nucleus is fixed. The electron, initially moving in an
r2
orbit of radius R1 jumps into an orbit of smaller radius R2. The decrease in the total energy of
the atom is.
ke2  1 1  ke2  R1 R2  ke2  1 1 ke2  R2 R 
(A)    (B)    (C)    (D)   
2  R1 R2  2  R22 R12  2  R 2 R1  2  R12 R22 

6. It is observed that some of the spectral lines in hydrogen spectrum have wavelengths almost
equal to those of the spectral lines in He+ ion, Out of the following the transitions in He+ that
will make this possible is
(A) n = 3 to n = 1 (B) n = 6 to n = 4 (C) n = 5 to n = 3 (D) n = 3 to n = 2
7. Find the smallest wavelength in emission spectra of (a) hydrogen, (b) He +
8. Calculate the angular frequency of revolution of an electron occupying the second Bohr orbit
of He+ ion.

DOWNLOAD OUR APP & GET : 353, Rajeev Gandhi Nagar, Instrumentation Limited Colony, Kota, Rajasthan 324005

UNLIMITED PRACTICE FOR FREE : www.atpstar.com PAGE NO.-129


PHYSICS

DPP
DAILY PRACTICE PROBLEMS

TARGET : JEE(MAIN + ADVANCED)


ST

NO. 7
MODERN PHYSICS
TOPIC : MP

1. Find the quantum number n corresponding to the excited state of He+ ion, if on transition to
the ground state that ion emits two photons in succession with wave lengths 108.5 and
30.4nm.
2. Consider a gas of hydrogen like ions in an excited state A. It emits photons having wavelength
equal to the wavelength of the first line of the Lyman series together with photons of five other
wavelengths. Identify the gas and find the principal quantum number of the state A.
3. A stationary hydrogen atom emits a photon corresponding to first line of the Lyman series.
What velocity does the atom acquire ?
4. From the condition of the foregoing problem, find how much (in %) the energy of the emitted
photon differs from the energy of the corresponding transition in a hydrogen atom.
5. Consider a gas consisting Li+2 (which is hydrogen like ion).
(a) Find the wavelength of radiation required to excite the electron in Li++ from n = 1 and n=3.
(Ionisation energy of the hydrogen atom equals 13.6 eV).
(b) How many spectral lines are observed in the emission spectrum of the above excited
system?
6. A free atom of iron emits a photon of energy 6.4 keV. Then find the recoil kinetic energy of
the atom. (Take mass of iron atom = 9.3 × 10–26 kg).
7. Three photons coming from emission spectra of hydrogen sample are picked up. Their
energies are 12.1eV, 10.2eV and 1.9eV. These photons must come from
(A) a single atom (B) two atoms
(C) three atom (D) either two atoms or three atoms
8. In a hypothetical atom, if transition from n = 4 to n = 3 produces visible light then the possible
transition to obtain infrared radiation is :
(A) n = 5 to n = 3 (B) n = 4 to n = 2 (C) n = 3 to n = 1 (D) none of these
9. The ionization energy of hydrogen atom is 13.6 eV. Hydrogen atoms in the ground state are
excited by electromagnetic radiation of energy 12.1 eV. How many spectral lines will be
emitted by the hydrogen atoms?
(A) one (B) two (C) three (D) four

DOWNLOAD OUR APP & GET : 353, Rajeev Gandhi Nagar, Instrumentation Limited Colony, Kota, Rajasthan 324005

UNLIMITED PRACTICE FOR FREE : www.atpstar.com PAGE NO.-130


PH YSI CS

DPP
DAILY PRACTICE PROBLEMS

TARGET : JEE(MAIN + ADVANCED)


ST

NO. 8
MODERN PHYSICS
TOPIC : MP

1. Energy levels A, B and C of a certain atom correspond to increasing values of energy, i.e. EA <

EB < EC. If 1, 2 and 3 are the wavelengths of radiations corresponding to transitions C to B,

B to A and C to A respectively, which of the following relations is correct ?


1 2
(A) 3 = 1 + 2 (B) 3 = (C) 1 + 2 + 3 = 0 (D) 32 = 12 + 22
1   2

2. The wavelength of the first line in balmer series in the hydrogen spectrum is . What is the
wavelength of the second line :
20 3 5 3
(A) (B) (C) (D)
27 16 36 4

3. The frequency of the first line in Lyman series in the hydrogen spectrum is . What is the
frequency of the corresponding line in the spectrum of doubly ionized Lithium ?
(A)  (B) 3  (C) 9  (D) 27 

4. A sodium atom emits a photon of wavelength 590 nm and recoils with velocity v equal to

(A) 0.029 m/s (B) 0.048 m/s (C) 0.0023 m/s (D) data inadequate
5. Three photons coming from emission spectra of hydrogen sample are picked up. Their
energies are 12.1eV, 10.2eV and 1.9eV. These photons must come from
(A) a single atom (B) two atoms
(C) three atom (D) either two atoms or three atoms
6. In a hypothetical atom, if transition from n = 4 to n = 3 produces visible light then the possible
transition to obtain infrared radiation is :
(A) n = 5 to n = 3 (B) n = 4 to n = 2 (C) n = 3 to n = 1 (D) none of these
7. The ionization energy of hydrogen atom is 13.6 eV. Hydrogen atoms in the ground state are
excited by electromagnetic radiation of energy 12.1 eV. How many spectral lines will be
emitted by the hydrogen atoms?
(A) one (B) two (C) three (D) four

DOWNLOAD OUR APP & GET : 353, Rajeev Gandhi Nagar, Instrumentation Limited Colony, Kota, Rajasthan 324005

UNLIMITED PRACTICE FOR FREE : www.atpstar.com PAGE NO.-131


PHYSI CS

DPP
DAILY PRACTICE PROBLEMS

TARGET : JEE(MAIN + ADVANCED)


ST

NO. 9
MODERN PHYSICS
TOPIC : MP

1. The wavelength of the first line in balmer series in the hydrogen spectrum is . What is the
wavelength of the second line :
20 3 5 3
(A) (B) (C) (D)
27 16 36 4
2. The frequency of the first line in Lyman series in the hydrogen spectrum is . What is the
frequency of the corresponding line in the spectrum of doubly ionized Lithium ?
(A)  (B) 3  (C) 9  (D) 27 

3. A sodium atom emits a photon of wavelength 590 nm and recoils with velocity v equal to

(A) 0.029 m/s (B) 0.048 m/s (C) 0.0023 m/s (D) data inadequate
4. At what minimum kinetic energy must a hydrogen atom move for its inelastic headon collision
with another stationary hydrogen atom so that one of them emits a photon? Both atoms are
supposed to be in the ground state prior to the collision.
5. An electron with kinetic energy 10 eV is incident on a hydrogen atom in its ground state. The
collision
(A) must be elastic (B) may be partially elastic
(C) must be completely inelastic (D) may be completely inelastic
6. Find the cutoff wavelength for the continuous X-rays coming from an X-ray tube operating at
40 kV.
7. If the operating potential in an X -ray tube is increased by 0.1%, by what percentage does the
cutoff wavelength decrease?
8. On increasing the operating voltage in an x-ray tube to 1.5 times, the shortest wavelength
decreases by 26 pm. Find the original value of operating voltage.

DOWNLOAD OUR APP & GET : 353, Rajeev Gandhi Nagar, Instrumentation Limited Colony, Kota, Rajasthan 324005

UNLIMITED PRACTICE FOR FREE : www.atpstar.com PAGE NO.-132


PHYSI CS

DPP
DAILY PRACTICE PROBLEMS

TARGET : JEE(MAIN + ADVANCED)


ST

NO. 10
MODERN PHYSICS
TOPIC : MP

1. An X-ray tube operates at 20 kV. Suppose the electron converts 70% of its energy into a
photon at each collision. Find the lowest three wavelength emitted from the tube. Neglect the
energy imparted to the atom with which the electron collides.
2. Figure shows the variation of frequency of a characteristic x-ray and atomic number.
(i) Name the characteristic x-ray
(ii) Find the energy of photon emitted when this x-ray is emitted by a metal having z = 101.

3. Find the wavelength of the K line in copper (Z = 29), if the wave length of the K line in iron
(Z = 26) is known to be equal to 193 pm. (Take b = 1)
4. A hydrogen like atom (atomic number Z) is in a higher excited state of quantum number n.
This excited atom can make a transition to the first excited state by successively emitting two
photons of energies 10.20 eV & 17.00 eV respectively. Alternatively, the atom from the same
excited state can make a transition to the second excited state by sucessively emitting two
photons of energies 4.25 eV & 5.95 eV respectively. Determine the values of n & Z.
(Ionization energy of hydrogen atom = 13.6 eV)
5. Characteristic X-rays of frequency 4.2 × 1018 Hz are emitted from a metal due to transition
from L- to K-shell. Find the atomic number of the metal using Moseley’s law. Take Rydberg
constant R = 1.1 × 107 m–1.
6. Consider a photon of continuous X-ray coming from a Coolidge tube. Energy of photon comes
from
(A) the kinetic energy of the striking electron
(B) the kinetic energy of the free electrons of the target
(C) the kinetic energy of the ions of the target
(D) an atomic transition in the target
7. If the voltage across the filament is increased, the cutoff wavelength
(A) will increase (B) will decrease (C) will remain unchanged (D) will change
8. The characteristic X-ray spectrum is emitted due to transition of
(A) valence electrons of the atom
(B) inner electrons of the atom
(C) nucleus of the atom
(D) both, the inner electrons and the nucleus of the atom
DOWNLOAD OUR APP & GET : 353, Rajeev Gandhi Nagar, Instrumentation Limited Colony, Kota, Rajasthan 324005

UNLIMITED PRACTICE FOR FREE : www.atpstar.com PAGE NO.-133


PHYSI CS

DPP
DAILY PRACTICE PROBLEMS

TARGET : JEE(MAIN + ADVANCED)


ST

NO. 11
MODERN PHYSICS
TOPIC : MP
1. When ultraviolet light is incident on a photocell, its stopping potential is V0 and the maximum
kinetic energy of the photoelectrons is Kmax. When X-rays are incident on the same cell, then :
(A) V0 and Kmax both increase (B) V0 and Kmax both decrease
(C) V0 increases but Kmax remains the same (D) Kmax increases but V0 remains the same
2. An electron beam of energy 10 KeV is incident on metallic foil. If the interatomic distance is
0.55Å . Find the angle of diffraction.
3. In Davisson-Germer experiment, the filament emits
(A) Photons (B) Protons (C) X-rays (D) Electrons
4. In the Davisson and Germer experiment, the velocity of electrons emitted from the electron
gun can be increased by :
(A) increasing the potential difference between the anode and filament
(B) increasing the filament current
(C) decreasing the filament current
(D) decreasing the potential difference between the anode and filament
5. The photoelectrons emitted from a metal surface :
(A) Are all at rest
(B) Have the same kinetic energy
(C) Have the same momentum
(D) Have speeds varying from zero up to a certain maximum value
6. A point source causes photoelectric effect from a small metal plate. Which of the following
curves may represent the saturation photocurrent as a function of the distance between the
source and the metal?

(A) (B) (C) (D)

7. In a photoelectric experiment, with light of wavelength , the fastest electron has speed v. If
3
the exciting wavelength is changed to , the speed of the fastest emitted electron will
4
become
3 4 3 4
(A) v (B) v 3 (C) less than v (D) greater than v 3
4 4
8. In a photoelectric experiment, the frequency and intensity of a light source are both doubled.
Then consider the following statements.
(i) The saturation photocurrent remains almost the same.
(ii) The maximum kinetic energy of the photoelectrons is doubled.
(A) Both (i) and (ii) are true (B) (i) is true but (ii) is false
(C) (i) is false but (ii) is true (D) both (i) and (ii) are false

DOWNLOAD OUR APP & GET : 353, Rajeev Gandhi Nagar, Instrumentation Limited Colony, Kota, Rajasthan 324005

UNLIMITED PRACTICE FOR FREE : www.atpstar.com PAGE NO.-134


PHYSICS

DPP
DAILY PRACTICE PROBLEMS

TARGET : JEE(MAIN + ADVANCED)


ST

NO. 12
MODERN PHYSICS
TOPIC : MP

1. When a monochromatic point source of light is at a distance of 0.2 m from a photoelectric cell,
the cut-off voltage and the saturation current are respectively 0.6 V and 18 mA. If the same
source is placed 0.6 m away from the cell, then :
(A) the stopping potential will be 0.2 V (B) the stopping potential will be 1.8 V
(C) the saturation current will be 6.0 mA (D) the saturation current will be 2.0 mA
2. An image of the sun is formed by a lens of focal length 30 cm on the metal surface of a photo-
electric cell and it produces a current . The lens forming the image is then replaced by
another lens of the same diameter but of focal length 15 cm. The photoelectric current in this
case will be : (In both cases the plate is kept at focal plane and normal to the axis lens).
(Assume saturation current only).
(A) /2 (B) 2  (C)  (D) 4 
hC
3. The work function of a certain metal is . When a monochromatic light of wavelength  < 0
0
is incident such that the plate gains a total power P. If the efficiency of photoelectric emission
is % and all the emitted photoelectrons are captured by a hollow conducting sphere of radius
R already charged to potential V, then neglecting any interaction between plate and the
sphere, expression of potential of the sphere at time t is (e= 1.6×10–19 C ) :

100 Pe t  Pe t Pe t


(A) V + (B) V – (C) V (D)
40RhC 4000RhC 40RhC
4. A neutron star has a density equal to that of the nuclear matter(3 × 1017 kg/m3). Assuming
the star to be spherical, find the radius of a neutron star whose mass is (i) 4.0 × 10 30 kg (twice
the mass of the sun) (ii) 6 × 1024 Kg (around mass of the earth).
5. Assuming the radius of a nucleus to be equal to R = 1.3 A1/3 × 10–15 m, where A is its mass
number, evaluate the density of nuclei and the number of nucleons per unit volume of the
nucleus. Take mass of one nucleon = 1.67 × 10–27 kg
6. The mass number of a nucleus is
(A) always less than its atomic number
(B) always more than its atomic number
(C) equal to its atomic number
(D) sometimes more than and sometimes equal to its atomic number
7. The stable nucleus that has a radius 1/3 that of Os189 is -
(A) 3Li7 (B) 2He4 (C) 5B10 (D) 6C12
8. The graph of n (R/R0) versus n A (R = radius of a nucleus and A = its mass number) is
(A) a straight line (B) a parabola (C) an ellipse (D) none of them

DOWNLOAD OUR APP & GET : 353, Rajeev Gandhi Nagar, Instrumentation Limited Colony, Kota, Rajasthan 324005

UNLIMITED PRACTICE FOR FREE : www.atpstar.com PAGE NO.-135


PHYSICS

DPP
DAILY PRACTICE PROBLEMS

TARGET : JEE(MAIN + ADVANCED)


ST

NO. 13
MODERN PHYSICS
TOPIC : MP

1. For uranium nucleus how does its mass vary with volume?

(A) m  V (B) m  1/V (C) m  V (D) m  V2


2. Let Fpp, Fpn and Fnn denote the magnitudes of the nuclear force by a proton on a proton, by a
proton on a neutron and by a neutron on a neutron respectively. When the separation is 1 fm,
(A) Fpp> Fpn = Fnn (B) Fpp= Fpn = Fnn (C) Fpp> Fpn > Fnn (D) Fpp< Fpn = Fnn
7
3. Find the binding energy of the nucleus of lithium isotope 3Li and hence find the binding
energy per nucleon in it. ( M 3 Li
7 = 7.014353 amu, M H = 1.007826, mass of neutron= 1.00867 u)
1
1

20
4. Find the energy required for separation of a 10Ne nucleus into two  particles and a 6C12
20
nucleus if it is known that the binding energies per nucleon in 10Ne , 2He4 and 6C12 nuclei are
equal to 8.03, 7.07 and 7.68 MeV respectively.
5. As the mass number A increases, the binding energy per nucleon in a nucleus
(A) increases
(B) decreases
(C) remains the same
(D) varies in a way that depends on the actual value of A.
6. Which of the following is a wrong description of binding energy of a nucleus ?
(A) It is the energy required to break a nucleus into its constituent nucleons.
(B) It is the energy released when free nucleons combine to from a nucleus
(C) It is the sum of the rest mass energies of its nucleons minus the rest mass energy of the
nucleus
(D) It is the sum of the kinetic energy of all the nucleons in the nucleus
7. The energy of the reaction Li7 + p  2 He4 is (the binding energy per nucleon in Li7 and He4
nuclei are 5.60 and 7.06 MeV respectively.)
(A) 17.3 MeV (B) 1.73 MeV
(C) 1.46 MeV (D) depends on binding energy of proton
10 11
8. The atomic weight of boron is 10.81 g/mole and it has two isotopes 5 B and 5 B . The ratio (by
10 10
number) of 5 B : 5 B in nature would be :

(A) 19 : 81 (B) 10 : 11 (C) 15 : 16 (D) 81 : 19

DOWNLOAD OUR APP & GET : 353, Rajeev Gandhi Nagar, Instrumentation Limited Colony, Kota, Rajasthan 324005

UNLIMITED PRACTICE FOR FREE : www.atpstar.com PAGE NO.-136


PHYSI CS

DPP
DAILY PRACTICE PROBLEMS

TARGET : JEE(MAIN + ADVANCED)


ST

NO. 14
MODERN PHYSICS
TOPIC : MP

1. The kinetic energy of an   particle which flies out of the nucleus of a Ra226 atom in
radioactive disintegration is 4.78 MeV. Find the total energy evolved during the escape of the
 particle.
64 64
2. In the decay Cu  Ni + e + + , the maximum kinetic energy carried by the positron is
found to be 0.680 MeV (a) Find the energy of the neutrino which was emitted together with a
positron of energy 0.180 MeV (b) What is the momentum of this neutrino in kgm /s ?Use the
formula applicable to photon.
3. Which of the following processes represents a gamma decay?
(A) AXZ +   AXZ  1 + a + b (B) AXZ + 1n0  A  3XZ  2 + c
(C) AXZ AXZ + f (D) AXZ + e1  AXZ  1 + g
14 17
4. An -particle is bombarded on N. As a result, a O nucleus is formed and a particle is
emitted. This particle is a
5. A free neutron decays into a proton, an electron and :
(A) A neutrino (B) An antineutrino (C) An -particle (D) A -particle ,

6. Nuclei X decay into nuclei Y by emitting  particles. Energies of  particle are found to be only
1 MeV & 1.4 MeV. Disregarding the recoil of nuclei Y. The energy of  photon emitted will be
(A) 0.8 MeV (B) 1.4 MeV (C) 1 MeV (D) 0.4 MeV
7. Beta decay of a free neutron takes place with a half life of 14 minutes. Then find (a) decay
constant (b) energy liberated in the process.

DOWNLOAD OUR APP & GET : 353, Rajeev Gandhi Nagar, Instrumentation Limited Colony, Kota, Rajasthan 324005

UNLIMITED PRACTICE FOR FREE : www.atpstar.com PAGE NO.-137


PHYSI CS

DPP
DAILY PRACTICE PROBLEMS

TARGET : JEE(MAIN + ADVANCED)


ST

NO. 15
MODERN PHYSICS
TOPIC : MP

1. How many  particles are emitted during one hour by 1.0 g of Na24 radionuclide whose
halflife is 15 hours? [Take e( 0.693/15) = 0.955, and Avagadro number = 6 × 1023 ]
2. Calculate the specific activities of Na24 & U235 nuclides whose half lives are 15 hours and
7.1 × 108 years respectively.
3. In one average-life ,
(A) half the active nuclei decay (B) less than half the active nuclei decay
(C) more than half the active nuclei decay (D) all the nuclei decay
4. A freshly prepared radioactive source of half-life 2 h emits radiation of intensity which is 64
times the permissible safe level. The minimum time after which it would be possible to work
safely with this source is -
(A) 6 h (B) 12 h (C) 24 h (D) 128 h
5. Two isotopes P and Q of atomic weight 10 and 20, respectively are mixed in equal amount by
weight. After 20 days their weight ratio is found to be 1 : 4. Isotope P has a half-life of 10 days.
The half-life of isotope Q is
(A) zero (B) 5 days (C) 20 days (D) inifinite
57
6. 10 grams of Co kept in an open container beta-decays with a half-life of 270 days. The
weight of the material inside the container after 540 days will be very nearly -
(A) 10 g (B) 7.5 g (C) 5 g (D) 2.5 g
 2
7. A  B  C
t=0 N0 0 0
t N1 N2 N3
The ratio of N1 to N2 when N2 is maximum is :
(A) at no time this is possible (B) 2
n2
(C) 1/2 (D)
2
8. The half-life of 131 is 8 days. Given a sample of 131 at time t = 0, we can assert that
(A) No nucleus will decay before t = 4 days
(B) No nucleus will decay before t = 8 days
(C) All nuclei will decay before t = 16 days
(D) A given nucleus may decay at any time after t = 0.

DOWNLOAD OUR APP & GET : 353, Rajeev Gandhi Nagar, Instrumentation Limited Colony, Kota, Rajasthan 324005

UNLIMITED PRACTICE FOR FREE : www.atpstar.com PAGE NO.-138


PHYSI CS

DPP
DAILY PRACTICE PROBLEMS

TARGET : JEE(MAIN + ADVANCED)


ST

NO. 16
MODERN PHYSICS
TOPIC : MP

1. Consider the case of bombardment of U235 nucleus with a thermal neutron. The fission
products are Mo95 & La139 and two neutrons. Calculate the energy released by one U235
nucleus. (Rest masses of the nuclides are U235 = 235.0439 u, 10 n = 1.0087 u,
Mo95 = 94.9058u, La139 = 138.9061 u).
2. Energy evolved from the fusion reaction 2 21 H = 42 He  Q is to be used for the production of
power. Assuming the efficiency of the process to be 30 %. Find the mass of deuterium that
will be consumed in a second for an output of 50 MW. 2He4 = 4.002603 amu and
2
1H = 2.014102 amu.
3. For the DT fusion reaction, find the rate at which deuterium & tritium are consumed to
produce 1 MW. The Qvalue of DT reaction is 17.6 MeV & assume all the energy from the
fusion reaction is available.
235
4. 92U nucleus absorbs a slow neutron and undergoes fission into 54X139 and 38Sr94 nuclei. The
other particles produced in this fission process are
(A) 1  and 1  (B) 2  and 1 neutron (C) 2 neturons (D) 3 neutrons
6
5. Two lithium Li nuclei in a lithium vapour at room temperature do not combine to form a
carbon 12C nucleus because
(A) a lithium nucleus is more tightly bound than a carbon nucleus
(B) carbon nucleus is an unstable particle
(C) it is not energetically favourable
(D) Coulomb repulsion does not allow the nuclei to come very close
6. In a uranium reactor whose thermal power is P = 100 MW, if the average number of neutrons
liberated in each nuclear splitting is 2.5. Each splitting is assumed to release an energy E =
200 MeV. The number of neutrons generated per unit time is -
125
(A) 4 × 1018 s–1 (B) 8 × 1023 s–1 (C) 8 × 1019 s–1 (D) × 1018 s–1
16
7. Choose the statement which is true.
(A) The energy released per unit mass is more in fission than in fusion
(B) The energy released per atom is more in fusion than in fission.
(C) The energy released per unit mass is more in fusion and that per atom is more in fission.
(D) Both fission and fusion produce same amount of energy per atom as well as per unit
mass.
8. Fusion reaction is possible at high temperature because -
(A) atoms are ionised at high temperature
(B) molecules break-up at high temperature
(C) nuclei break-up at high temperature
(D) kinetic energy is high enough to overcome repulsion between nuclei.

DOWNLOAD OUR APP & GET : 353, Rajeev Gandhi Nagar, Instrumentation Limited Colony, Kota, Rajasthan 324005

UNLIMITED PRACTICE FOR FREE : www.atpstar.com PAGE NO.-139


PHYSI CS

DPP
DAILY PRACTICE PROBLEMS

TARGET : JEE(MAIN + ADVANCED)


ST

NO. 17
MODERN PHYSICS
TOPIC : MP
236 117 117
1. In a fission reaction 92 U  X+ Y + n + n the average binding energy per nucleon of
236
X and Y is 8.5 MeV whereas that of U is 7.6 MeV. The total energy liberated will be about :
(A) 200 keV (B) 2 MeV (C) 200 MeV (D) 2000 MeV
2. A heavy nucleus having mass number 200 gets disintegrated into two small fragments of
mass number 80 and 120. If binding energy per nucleon for parent atom is 6.5 M eV and for
daughter nuclei is 7 M eV and 8 M eV respectively, then the energy released in each decay
will be :
(A) 200 M eV (B)  220 M eV (C) 220 M eV (D) 180 M eV
3. Assuming that about 20 MeV of energy is released per fusion reaction, 1H2 + 1H3  0n1 + 2He4,
the mass of 1H2 consumed per day in a future fusion reactor of power 1 MW would be
approximately
(A) 0.1 gm (B) 0.01 gm (C) 1 gm (D) 10 gm
4. Choose the wrong statement.
(A) The nuclear force becomes weak if the nucleus contains too many protons compared to
the number of neutrons
(B) The nuclear force becomes weak if the nucleus contains too many neutrons compared to
the number of protons.
(C) Nuclei with atomic number greater than 82 show a tendency to disintegrate.
(D) The nuclear force becomes very strong if the nucleus contains a large number of
nucleons.
5. Binding Energy per nucleon of a fixed nucleus XA is 6 MeV. It absorbs a neutron moving with
KE = 2 MeV, and converts into Y at ground state, emitting a photon of energy 1 MeV. The
Binding Energy per nucleon of Y (in MeV) is -
(6A  1) (6A  1) 7
(A) (B) (C) 7 (D)
(A  1) (A  1) 6
6. The half life of 215At is 100 s. The time taken for the radioactivity of a sample of 215At to decay
to 1/16th of its initial value is :
(A) 400 s (B) 6.3 s (C) 40 s (D) 300 s
7. A free neutron decays to a proton but a free proton does not decay to a neutron. This is
because
(A) neutron is a composite particle made of a proton and an electron whereas proton is
fundamental particle
(B) neutron is an uncharged particle whereas proton is a charged particle
(C) neutron has larger rest mass than the proton
(D) weak forces can operate in a neutron but not in a proton.

DOWNLOAD OUR APP & GET : 353, Rajeev Gandhi Nagar, Instrumentation Limited Colony, Kota, Rajasthan 324005

UNLIMITED PRACTICE FOR FREE : www.atpstar.com PAGE NO.-140


8. Match the following :
Column  Column 
(a) Photoelectric effect . Photon
(b) Wave . Frequency
(c) X rays . K capture
(d) Nucleus V.  rays
(A) a – , b – , c – , d – V
(B) a – , b – , c – V, d – 
(C) a – , b – , c – , d – V
(D) None of these

9. Protons and singly ionized atoms of U235 & U238 are passed in turn (which means one after the
other and not at the same time) through a velocity selector and then enter a uniform magnetic
field. The protons describe semicircles of radius 10 mm. The separation between the ions of
U235 and U238 after describing semicircle is given by
U-238 U-235 p

(A) 60 mm (B) 30 mm (C) 2350 mm (D) 2380 mm

DOWNLOAD OUR APP & GET : 353, Rajeev Gandhi Nagar, Instrumentation Limited Colony, Kota, Rajasthan 324005

UNLIMITED PRACTICE FOR FREE : www.atpstar.com PAGE NO.-141


PHYSI CS

DPP
DAILY PRACTICE PROBLEMS

TARGET : JEE(MAIN + ADVANCED)


ST

NO. 18
MODERN PHYSICS
TOPIC : MP

1. When a  -particle is emitted from a nucleus, the neutron-proton ratio:
(A) is decreased (B) is increased
(C) remains the same (D) first (A) then (B)
2. Consider a sample of a pure beta-active material
(A) All the beta particles emitted have the same energy
(B) The beta particles originally exist inside the nucleus and are ejected at the time of beta
decay
(C) The antineutrino emitted in a beta decay has zero rest mass and hence zero momentum
(D) The active nucleus changes to one of its isobars after the beta decay
64 64
3. Masses of two isobars 29 Cu and 30 Zn are 63.9298 u and 63.9292 u respectively. It can be
concluded from these data that :
(A) Both the isobars are stable
64 64
(B) Zn is radioactive, decaying to Cu through -decay
64 64
(C) Cu is radioactive, decaying to Zn through -decay
64 64
(D) Cu is radioactive, decaying to Zn through -decay
4. In an -decay the Kinetic energy of  particle is 48 MeV and Q-value of the reaction is 50
MeV. The mass number of the mother nucleus is:- (Assume that daughter nucleus is in
ground state)
(A) 96 (B) 100 (C) 104 (D) none of these
5. Free 238U nuclei kept in a train emit alpha particles. When the train is stationary and a uranium
nucleus decays, a passenger measures that the separation between the alpha particle and
the recoiling nucleus becomes x in time t after the decay. If a decay takes place when the
train is moving at a uniform speed , the distance between the alpha particle and the recoiling
nucleus at a time t after the decay, as measured by the passenger will be
(A) x + t (B) x - t
(C) x (D) depends on the direction of the train
6. A nucleus with mass number 220 initially at rest emits an -particle. If the Q value of the
reaction is 5.5 MeV, calculate the kinetic energy of the -particle
(A) 4.4 MeV (B) 5.4 MeV (C) 5.6 MeV (D) 6.5 MeV
7. A charged capacitor of capacitance C is discharged through a resistance R. A radioactive
sample decays with an average life  Find the value of R for which the ratio of the
electrostatic field energy stored in the capacitor to the activity of the radioactive sample is
independent of time.
 2  3
(A) (B) (C) (D)
C C 2C 2C

DOWNLOAD OUR APP & GET : 353, Rajeev Gandhi Nagar, Instrumentation Limited Colony, Kota, Rajasthan 324005

UNLIMITED PRACTICE FOR FREE : www.atpstar.com PAGE NO.-142


8. At time t = 0, some radioactive gas is injected into a sealed vessel. At time T, some more of
the same gas is injected into the same vessel. Which one of the following graphs best
represents the variation of the logarithm of the activity A of the gas with time t ?

(A) (B)

(C) (D)

DOWNLOAD OUR APP & GET : 353, Rajeev Gandhi Nagar, Instrumentation Limited Colony, Kota, Rajasthan 324005

UNLIMITED PRACTICE FOR FREE : www.atpstar.com PAGE NO.-143


PHYSI CS

DPP
DAILY PRACTICE PROBLEMS

TARGET : JEE(MAIN + ADVANCED)


ST

NO. 19
MODERN PHYSICS
TOPIC : MP

1. A sample of radioactive material has mass m, decay constant , and molecular weight M.
Avogadro constant = NA. The initial acitvity of the sample is :
m  m NA
(A) m (B) (C) (D) mNAe
M M
2. Two radioactive sources A and B initially contain equal number of radioactive atoms. Source
A has a half-life of 1 hour and source B has a half-life of 2 hours. At the end of 2 hours, the
ratio of the rate of disintegration of A to that of B is :
(A) 1 : 2 (B) 2 : 1 (C) 1 : 1 (D) 1 : 4
3. Two identical samples (same material and same amount initially) P and Q of a radioactive
substance having mean life T are observed to have activities AP & AQ respectively at the time
of observation. If P is older than Q, then the difference in their ages is:
 AP   AQ  A   AP 
(A) T n   (B) T n   (C) 1 n  P  (D) T  
 AQ   AP  T  AQ   AQ 
4. N atoms of a radioactive element emit n alpha particles per second at an instant. Then the
half - life of the element is
n n n N
(A) sec. (B) 1.44 sec. (C) 0.69 sec. (D) 0.69 sec.
N N N n
5. The radioactivity of an old sample of a liquid due to tritium (half life 12.5 years) was found to
be only about 3% of that measured in a recently purchased bottle marked ‘7 year old’. The
sample must have been prepared about :
(A) 70 year (B) 220 year (C) 420 year (D) 300 year
1 2
6. A   B   C
t = 0 N0 0 0
t N1 N2 N3
In the above radioactive decay C is stable nucleus. Then:
(A) rate of decay of A will first increase and then decrease
(B) number of nuclei of B will first increase and then decrease
(C) if 2 > 1, then activity of B will always be higher than activity of A
(D) if 1 >> 2, then number of nucleus of C will always be less than number of nucleus of B.
7. Ninety percent of a radioactive sample is left over after a time interval t. The percentage of
initial sample that will disintegrate in an interval 2t is
(A) 38% (B) 19% (C) 9% (D) 62%
8. The intensity of gamma radiation from a given source is . On passing through 36 mm of lead,
it is reduced to 1/8. The thickness of lead, which will reduce the intensity to 1/2 will be :
(A) 6 mm (B) 9 mm (C) 18 mm (D) 12 mm
9. The fraction of the original number of nuclei of a radioactive atom having a mean life of 10
days, that decays during the 5th day is :
(A) 0.15 (B) 0.30 (C) 0.045 (D) 0.064

DOWNLOAD OUR APP & GET : 353, Rajeev Gandhi Nagar, Instrumentation Limited Colony, Kota, Rajasthan 324005

UNLIMITED PRACTICE FOR FREE : www.atpstar.com PAGE NO.-144


ANSWER KEY
DPP – 1
2 4
P  1.24  10  – 25
1.   2.5  eV  0.6 eV ; P = 2  9.1 10 31  0.6  1.6  10 19 = 4.2 ×10 kg.m/s
2m  4000 
2. (0.6  1015 h  2e) J = 0.48 eV
P e
3. =.  = 1.84  106 amp 
E 100
hc d hc
4. dVs = .  × 107  5.5  102 volt
e 2 228e
 9  1015 
5.  h  2  eV = 3.93 eV
 2e 
P
6. e A = 3.2 µ A
hc  106

7.

8. (B)

DPP – 2
1. (C) 2. (A) 3. (A) 4. (B) 5. (B)
6. (B) 7. (B) 8. (D)

DPP – 3
7  104 21
1. (a) N = = 3.5 × 10 ,
hc
7  104 13
(b) = 1.2 × 10 ,
hc 2
7  104  4(1.5  1011 )2 44
(c) =9.9 × 10
hc
2. 5 × 10–8 N
1.3  30
3.  1.3  10 7 N
3  108
354  108
4. = 1.77 × 1019
hc
5. (D)
h.. th 6  107 h
6. d = = m = 12.08 Å
2m.c.( th   ) mec

21 2
7. = 2

1   22
8. (B)

DOWNLOAD OUR APP & GET : 353, Rajeev Gandhi Nagar, Instrumentation Limited Colony, Kota, Rajasthan 324005
UNLIMITED PRACTICE FOR FREE
: www.atpstar.com PAGE NO.-158
DPP – 4
1. (D) 2. (C) 3. (A) 4. (C) 5. (D)
6.  = 2r = 2 × 0.529 Å = 1.058  Å 
22
7. (a) r = 0.529 × = 1.058 Å
2
22
E = –13.6 × 2 = – 13.6 eV
2
32
(b) r = 0.529 × = 2.38 Å
2
22
E = – 13.6 × 2 = – 6.04 eV.
3
+1
8. He
2E0 5
9. K= T = 1.05 × 10 K
3K

DPP – 5
10
6 hc  10  15
1. = 13.6e   = 5×10 Hz,
h 1215 
2. (B) 3. (B) 4. (C) 5. (B) 6. (A)
7. (D)

DPP – 6
1 (B) 2. (B) 3. (C) 4. (D) 5. (C)
6. (B)
7. (a) 91 nm (b) 23 nm
v 0 z2 2.19  106 Z2 16 1
8. . 3 =  3 = 2.07 × 10 s
r0 n 0.529  10  10
n
DPP – 7
1. n=5
2. He + 4,
E 1 13.6  3e
3. . = 4cm = 3.25 m/s
c m p

4.
(E - E) × 100 = 0.55 x 106 %
E
hc
5. (a) = 113.7 Å (b) 3
13.6  8e
2
 6.4  103 e  1 –4
6. Krecoil =    26
J = 3.9 × 10 eV
 c  2  (9.3  10 )
7. (D) 8. (D) 9. (C)

DPP – 8
1. (B) 2. (A) 3. (C) 4. (A)

DPP – 9
T
1. (A) 2. (C) 3. (A) 4. Tmin = = 10.2 = 20.4 eV
2
hc
5. (A) 6. = m = 31.05 pm 7. approximately 0.1%
40  103 e
hc
8. v1 = V = 15.9 kV
1.5e(1  26  1012 )

DOWNLOAD OUR APP & GET : 353, Rajeev Gandhi Nagar, Instrumentation Limited Colony, Kota, Rajasthan 324005
UNLIMITED PRACTICE FOR FREE
: www.atpstar.com PAGE NO.-159
DPP – 10
hc hc
1. 1 = = 88.6 pm , 2 = = 295.6 pm, 
20  103  (0.7)e 20  103  (0.7  0.3)e
hc
 3 = = 985.6 pm,
20  10  0.7  (0.3)2 e
3

2. (i) k (ii) 102 keV.


2
 26  1 
3. =   193 pm = 154 pm 
 29  1 
4. n = 6, Z = 3
5. (42) 6. (A) 7. (C) 8. (B)

DPP – 11
1. (A) 3. (D) 4. (A) 5. (D) 6. (D)
7. (D) 8. (B)

DPP – 12
1. (D) 2. (C) 3. (B)
1/ 3 1/ 3
30
 4  10 3   6  1024 3 
4. (i) r1 =  17
  = 14.71 km (ii) r2 =  17
  = 168.4 m
 3  10 4   3  10 4 
5. 2 ×1011 kg/cm3, 1×1038 nucl. /cm3
6. (D) 7. (A) 8. (A)

DPP – 13
1. (A) 2. (B)
B.E. 39.22
3. B.E. = 3M H  4m
1
1
0n
1 –M
3 Li
7
 931 MeV = 39.22 MeV ,
 A

7 = 5.6 MeV
4. E = 20 × (8.03) – 2×4 (7.07) – 12(7.68) = 11.9 MeV
5. (D) 6. (D) 7. (A) 8. (A)

DPP – 14
226
1.  4.78 = 4.87 MeV.
222
500  103 e
2. (a) (0.680 – 0.180) Me V = 500 ke V (b) = 2.67 × 10 – 22 kg–m/s
C
3. (C)
4 14 17
4. 2 He + 7 N  8 O + 11H
5. (B)
6. (D)
0.693
7. (a) = 8.25 × 10– 4 s– 1 (b) (mn – mp – me) 931 = 782 keV
14  60

DPP – 15
6  1023  10 6
1. 1  e 0.693 / 15  = 1.128 × 1015
24
NA 0.693 NA 0.693
2.  = 3.2 × 1017 dps &  = 0.8 × 105 dps
24 15  60  60 235 7.1 108  365  86400
3. (C) 4. (B) 5. (D) 6. (A) 7. (B)
8. (D)

DOWNLOAD OUR APP & GET : 353, Rajeev Gandhi Nagar, Instrumentation Limited Colony, Kota, Rajasthan 324005
UNLIMITED PRACTICE FOR FREE
: www.atpstar.com PAGE NO.-160
DPP – 16
1. 207.9 MeV. 2. 23.8 MeV. 3. 1.769 × 109 kg/s
4. (D) 5. (D) 6. (D) 7. (C) 8. (D)

DPP – 17
1. (C) 2. (C) 3. (A) 4. (D) 5. (B)
6. (A) 7. (C) 8. (A) 9. (A)

DPP – 18
1. (A) 2. (D) 3. (D) 4. (B) 5. (C)
6. (B) 7. (B) 8. (B)

DPP – 19
1. (C) 2. (C) 3. (B) 4. (D) 5. (A)
6. (B) 7. (B) 8. (D) 9. (D)

DOWNLOAD OUR APP & GET : 353, Rajeev Gandhi Nagar, Instrumentation Limited Colony, Kota, Rajasthan 324005
UNLIMITED PRACTICE FOR FREE
: www.atpstar.com PAGE NO.-161
PHYSI CS

DPP
DAILY PRACTICE PROBLEMS

TARGET : JEE(MAIN + ADVANCED)


ST

NO. 1
WAVE OPTICS
TOPIC : YE

1. Two sources of intensity  & 4 are used in an interference experiment. Find the intensity at
points where the waves from the two sources superimpose with a phase difference of

(a) zero (b) & (c) They meet at phase difference of .
2
2. An electromagnetic wave travelling through a transparent medium is given by
 y 
E x (y,t)  Eox sin2  7
 3  1014 t  in S units. Then what is the refractive index of the
 5  10 
medium?
3. Ratio of intensities of two light waves is given by 4 : 1. The ratio of the amplitudes of the
waves is :
(A) 2 : 1 (B) 1 : 2 (C) 4 : 1 (D) 1 : 4
4. Two coherent monochromatic light beams of intensities I and 4I are superimposed; the
maximum and minimum possible intensities in the resulting beam are :
(A) 5 and  (B) 5 and 3 (C) 9 and  (D) 9 and 3
5. What is the effect on the fringe width of interference fringes in a Young's double slit
experiment due to each of the following operations.
(a) The screen is moved away from the plane of the slits.
(b) the (monochromatic) source is replaced by another (monochromatic) source of shorter
wavelength.
(c) The separation between the two slits is increased.
(d) The width of two slits are slightly increased.
[In each operation, take all parameters, other than the one specified to remain
unchanged]
6. Two slits separated by a distance of 1 mm, are illuminated with red light of wavelength 6.5 ×
10–7 m. The interference fringes are observed on a screen placed 1 m from the slits. Find the
distance between the third dark fringe and the fifth bright fringe on the same side of the
central maxima.
7. In a Young’s double slit experiment, the fringe width is found to be 0.4 mm. If the whole
apparatus is immersed in water of refractive index (4/3), without disturbing the geometrical
arrangement, what is the new fringe width?
8. Find the angular fringe width in a Young’s double slits experiment with blue-green light of
wavelength 6000 Å. The separation between the slits is 3.0 × 10–3 m.

DOWNLOAD OUR APP & GET : 353, Rajeev Gandhi Nagar, Instrumentation Limited Colony, Kota, Rajasthan 324005

UNLIMITED PRACTICE FOR FREE : www.atpstar.com PAGE NO.-164


PHYSI CS

DPP
DAILY PRACTICE PROBLEMS

TARGET : JEE(MAIN + ADVANCED)


ST

NO. 2
WAVE OPTICS
TOPIC : YE

1. The contrast in the fringes in any interference pattern depends on :


(A) Fringe width (B) Wavelength
(C) Intensity ratio of the sources (D) Distance between the sources
2. Initially interference is observed with the entire experimental set up inside a chamber filled
with air, Now the chamber is evacuated. With the same source of light used, a careful
observer will find that
(A) The interference pattern is almost absent as it is very much diffused
(B) There is no change in the interference pattern
(C) The fringe width is slightly decreased
(D) The fringe width is slightly increased
3. Yellow light emitted by sodium lamp in Young’s double slit experiment is replaced by
monochromatic blue light of the same intensity :
(A) fringe width will decrease. (B) fringe width will increase.
(C) fringe width will remain unchanged. (D) fringes will become less intense.
4. In a YDSE: D = 1 m, d = 1 mm and  = 500 n m. The distance of 1000th maxima from the
central maxima is:
(A) 0.5 m (B) 0.577 m (C) 0.495 m (D) does not exist
5. In a Young's double slit experiment, d = 1 mm,  = 6000 Å & D = 1 m. The slits produce same
intensity on the screen. The minimum distance between two points on the screen having 75 %
intensity of the maximum intensity is:
(A) 0.45 mm (B) 0.40 mm (C) 0.30 mm (D) 0.20mm
6. Two coherent light sources each of wavelength  are separated by a distance 3 . The total
number of minima formed on line AB which runs from -  to + is:

(A) 2 (B) 4 (C) 6 (D) 8

DOWNLOAD OUR APP & GET : 353, Rajeev Gandhi Nagar, Instrumentation Limited Colony, Kota, Rajasthan 324005

UNLIMITED PRACTICE FOR FREE : www.atpstar.com PAGE NO.-165


7. In a Young’s double slit experiment the intensity at a point  where the corresponding path
difference is one sixth of the wavelength of light used. If 0 denotes the maximum intensity, the

ratio  is equal to
0

1 1 3 3
(A) (B) (C) (D)
4 2 2 4
8. A source emitting two light waves of wavelengths 580 nm and 700 nm is used in a young's
double slit interference experiment. The separation between the slits is 0.20 mm and the
interference is observed on a screen placed at 150 cm from the slits. Find the linear
separation between the first maximum (next to the central maximum) corresponding to the two
wavelengths.

DOWNLOAD OUR APP & GET : 353, Rajeev Gandhi Nagar, Instrumentation Limited Colony, Kota, Rajasthan 324005

UNLIMITED PRACTICE FOR FREE : www.atpstar.com PAGE NO.-166


PHYSI CS

DPP
DAILY PRACTICE PROBLEMS

TARGET : JEE(MAIN + ADVANCED)


ST

NO. 3
WAVE OPTICS
TOPIC : YE

1. In the figure shown if a parallel beam of white light is incident on the plane of the slits then the
distance of the nearest white spot on the screen from O is: [assume d << D,  << d ]

(A) 0 (B) d/2 (C) d/3 (D) d/6


2. The Young’s double slit experiment is performed with blue and with green light of wavelengths
4360 Å and 5460 Å respectively. If X is the distance of 4th maximum from the central one,
then :
(A) X(blue) = X(green) (B) X(blue) > X(green)
X(blue) 5460
(C) X(blue) < X(green) (D) X(green)  4360

3. A flint glass and a crown glass are fitted on the two slits of a double slit apparatus. The
thickness of the strips is 0.40 mm and the separation between the slits is 0.12 cm. The
refractive index of flint glass and crown glass are 1.62 and 1.52 respectively for the light of
wavelength 480 nm which is used in the experiment. The interference is observed on a screen
a distance one meter away. (a) What would be the fringe-width? (b) At what distance from the
geometrical centre will the nearest maximum be located?
4. Find the thickness of a plate which will produce a change in optical path equal to one fourth of
the wavelength  of the light passing through it normally. The refractive index of the plate is µ.
5. A two slit Young’s interference experiment is done with monochromatic light of wavelength
6000 Å. The slits are 2 mm apart. The fringes are observed on a screen placed 10 cm away
from the slits. Now a transparent plate of thickness 0.5 mm is placed in front of one of the slits
and it is found that the interference pattern shifts by 5 mm. The refractive index of the
transparent plate is :
(A)1.2 (B) 0.6 (C) 2.4 (D) 1.5
6. In a YDSE both slits produce equal intensities on the screen. A 100 % transparent thin film is
placed in front of one of the slits. Now the intensity of the geometrical centre of system on the
screen becomes 75 % of the previous intensity. The wavelength of the light is 6000Å and glass
= 1.5. The thickness of the film cannot be:
(A) 0.2  m (B) 1.0  m (C) 1.4  m (D) 1.6  m
DOWNLOAD OUR APP & GET : 353, Rajeev Gandhi Nagar, Instrumentation Limited Colony, Kota, Rajasthan 324005

UNLIMITED PRACTICE FOR FREE : www.atpstar.com PAGE NO.-167


7. A parallel beam of monochromatic light of wavelength  is used in a Young’s double slit
experiment. The siits are separated by a distance d and the screen is placed parallel to the
  
plane of the slits. The incident beam makes an angle  = sin–1   with the normal to the
 2d 


plane of the slits. A transparent sheet of refractive index. '' and thickness t = is
2(  1)

introduced in front of one of the slit. Find the intensity at the geometrical centre .
8. In a YDSE experiment, thin films of thickness t1 and t2 are placed in front of slits S1 and S2 as
shown in figure-1 and figure-2. It is observed that first minima and second maxima are
produced at point 'O' in first and second experiment respectively. Point 'O' and 'S' are
t2
symmetrical with respect to S1 and S2 . Both the films have same refractive index if = x ,
t1 25
then calculate 'x' :

(A) 3 (B) 9 (C) 12 (D) 15

DOWNLOAD OUR APP & GET : 353, Rajeev Gandhi Nagar, Instrumentation Limited Colony, Kota, Rajasthan 324005

UNLIMITED PRACTICE FOR FREE : www.atpstar.com PAGE NO.-168


PHYSI CS

DPP
DAILY PRACTICE PROBLEMS

TARGET : JEE(MAIN + ADVANCED)


ST

NO. 4
WAVE OPTICS
TOPIC : YE

1. In Youngs double slit experiment, distance between the slits is d and that between the slits
and screen is D. Angle between principle axis of lens and perpendicular bisector of S1 and S2
is 45°. The point source S is placed at the focus of lens and aperture of lens is much larger
than d. Assuming only the reflected light from plane mirror M is incident on slits, distance of
central maxima from O will be :

D D
(A) D (B) (C) D 3 (D)
3 4
2. A soap film of thickness 0.3 m appears dark when seen by the refracted light of wavelength
580 nm. What is the index of refraction of the soap solution, if it is known to be between 1.3
and 1.5?
3. A parallel beam of light of wavelength 560 nm falls on a thin film of oil (refractive index = 1.4).
What should be the minimum thickness of the film so that it weakly transmits the light?
4. White light is incident normally on a glass plate (in air) of thickness 500 nm and refractive
index of 1.5. The wavelength (in nm) in the visible region (400 nm - 700nm) that is strongly
reflected by the plate is:
(A) 450 (B) 600 (C) 400 (D) 500
0
5. A slit of width ‘a’ is illuminated by light of wavelength 6000 A For What value of ‘a will the :-
(i) First maximum fall at an angle of diffraction of 300 ?
(ii) First minimum fall at an angle of diffraction 300 ?
6. A slit of size 0.15 cm is placed at 2.1 m from a screen. On illuminating it by a light of
-5
wavelength 5 x 10 cm, the width of diffraction pattern will be:-
(A) 70 mm (B) 0.14 mm (C) 1.4 cm (D) 0.14 cm
7. The diameter of objective of a telescope is 1m. Its resolving limit for the light of wave length
4538 Å, will be
7 4 7
(A) 5.54  10 rad (B) 2.54 10 rad (C) 6.54 10 rad (D) None of these
8. When an unpolarized light of intensity I0 is incident on a polarizing sheet, the intensity of the
light which does not get transmitted is :
1 1
(A) I0 (B) I0 (C) zero (D) I0
2 4

DOWNLOAD OUR APP & GET : 353, Rajeev Gandhi Nagar, Instrumentation Limited Colony, Kota, Rajasthan 324005

UNLIMITED PRACTICE FOR FREE : www.atpstar.com PAGE NO.-169


PHYSI CS

DPP
DAILY PRACTICE PROBLEMS

TARGET : JEE(MAIN + ADVANCED)


ST

NO. 5
WAVE OPTICS
TOPIC : YE

1. A single slit diffraction pattern is obtained using a beam of red light. What happens if the red
light is replaced by blue light?
(A) There is no change in the diffraction pattern
(B) Diffraction fringes become narrower and crowded together
(C) Diffraction fringes become boarder and crowded together
(D) The diffraction pattern disappears
2. Shows a glass plate placed vertically on a horizontal table with a beam of unpolarised light
falling on its surface at 57° with the normal. The electric vectors in the reflected light on the
screen S will vibrate with respect to the plane of incidence :
M
S
57°

(A) in a vertical plane


(B) in a horizontal plane
(C) in a plane making an angle of 45° with the vertical
(D) in a plane making an angle of 57° with the horizontal
3. Two point white dots are 1 mm apart on a black paper. They are viewed by eye of pupil
diameter 3 mm. Approximately, what is the maximum distance at which these dots can be
resolved by the eye?
[Take wavelength of light = 500 nm]
(A) 6m (B) 3m (C) 5m (D) 1m
4. Visible light passing through a circular hole form a diffraction disc of radius 0.1 mm on a
screen. If X-ray is passed through the same set-up, the radius of the diffraction disc will be :
(A) zero (B) < 0.1 mm (C) 0.1 mm (D) > 0.1 mm
5. The resolving power of a telescope is more when its objective lens has
(A) greater focal length (B) smaller focal length
(C) greater diameter (D) smaller diameter
6. Resolving power of a microscope depends upon
(A) the focal length and aperture of the eye lens
(B) the focal lengths of the objective and the eye lens
(C) the apertures of the objective and the eye lens
(D) the wavelength of light illuminating the object
DOWNLOAD OUR APP & GET : 353, Rajeev Gandhi Nagar, Instrumentation Limited Colony, Kota, Rajasthan 324005

UNLIMITED PRACTICE FOR FREE : www.atpstar.com PAGE NO.-170


7. If the ratio of the intensity of two coherent sources is 4 then the visibility
[(max- min)/(max+ min)] of the fringes is
(A) 4 (B) 4/5 (C) 3/5 (D) 9
8. In a YDSE experiment if a slab whose refractive index can be varied is placed in front of one
of the slits then the variation of resultant intensity at mid-point of screen with 'µ' will be best
represented by (µ  1).[ Assume slits of equal width and there is no absorption by slab]

(A) (B)

(C) (D)

DOWNLOAD OUR APP & GET : 353, Rajeev Gandhi Nagar, Instrumentation Limited Colony, Kota, Rajasthan 324005

UNLIMITED PRACTICE FOR FREE : www.atpstar.com PAGE NO.-171


PHYSI CS

DPP
DAILY PRACTICE PROBLEMS

TARGET : JEE(MAIN + ADVANCED)


ST

NO. 6
WAVE OPTICS
TOPIC : YE

1. In a Young’s double slit experiment the slit is illuminated by a source having two wavelengths
of 400 nm and 600 nm. If distance between slits, d = 1mm, and distance between the plane of
the slit and screen, D = 10 m then the smallest distance from the central maximum where
there is complete darkness is :
(A) 2mm (B) 3mm (C) 12 mm (D) there is no such point
2. If the first minima in a Young’s slit experiment occurs directly in front of one of the slits.
(distance between slit & screen D = 12 cm and distance between slits d = 5 cm) then the
wavelength of the radiation used is :
(A) 2 cm only (B) 4 cm only
2 2 4 4
(C) 2m, cm, cm (D) 4cm, cm, cm
3 5 3 5
3. An interference is observed due to two coherent sources 'A' & 'B' having phase constant zero
separated by a distance 4  along the y  axis where  is the wavelength of the source. A
detector D is moved on the positive x  axis. The number of points on the x  axis excluding
the points, x = 0 & x =  at which maximum will be observed is

(A) three (B) four (C) two (D) infinite


4. Two coherent sources of light S1 and S2, equidistant from the origin, are separated by a
distance 2 as shown. They emit light of wavelength . Interference is observed on a screen
placed along the circle of large radius R. Point is seen to be a point of constructive
interference. Then angle  (other than 0° and 90°) is

(A) 45° (B) 30°


(C) 60° (D) Not possible in the first quandrant

DOWNLOAD OUR APP & GET : 353, Rajeev Gandhi Nagar, Instrumentation Limited Colony, Kota, Rajasthan 324005

UNLIMITED PRACTICE FOR FREE : www.atpstar.com PAGE NO.-172


5. In the figure shown, a parallel beam of light is incident on the plane of the slits of a Young’s
double slit experiment. Light incident on the slit, S1 passes through a medium of variable
refractive index  = 1 + ax(where ‘x’ is the distance from the plane of slits as shown), upto a
distance ‘’ before falling on S1. Rest of the space is filled with air. If at ‘O’ a minima is formed,
then the minimum value of the positive constant a (in terms of  and wavelength ‘’ in air) is :

  2
(A) (B) (C) (D) None of these
 2 
6. M1 and M2 are two plane mirrors which are kept parallel to each other as shown. There is a
point 'O' on perpendicular screen just in front of 'S'. What should be the wavelength of light
coming from monochromatic source 'S'. So that a maxima is formed at 'O' due to interference
of reflected light from both the mirrors. [Consider only 1st reflection]. [D > > d, d > > ]

3d2 3d2 d2 2d2


(A) (B) (C) (D)
D 2D D D
7. A long narrow horizontal slit lies 1 mm above a plane mirror. The interference pattern
produced by the slit and its image is viewed on a screen distant 1m from the slit. The
wavelength of light is 600 nm. Then the distance of the first maxima above the mirror is equal
to (d << D):

S
d
0
D 
(A) 0.30 mm (B) 0.15 mm (C) 60 mm (D) 7.5 mm
8. A parallel monochromatic beam of light is incident normally on a narrow slit. A diffraction
pattern is formed on a screen placed perpendicular to the direction of the incident beam. At
the first minimum of the diffraction pattern, the phase difference between the rays coming
from the two edges of the slit is: [Diffraction – Not in JEE syllabus now]
(A) 0 (B) /2 (C)  (D) 2 
9. A slit of width a is illuminated by parallel monochromatic light of wavelength . The value of a
at which the first minimum of the diffraction pattern will from at  = 30° is
(A) /2 (B)  (C) 2 (D) 3

DOWNLOAD OUR APP & GET : 353, Rajeev Gandhi Nagar, Instrumentation Limited Colony, Kota, Rajasthan 324005

UNLIMITED PRACTICE FOR FREE : www.atpstar.com PAGE NO.-173


ANSWER KEY
DPP – 1
1. (a) 9  (b) 5  (c)  2. (2) 3. (A) 4. (C)
5. (a) Angular separation of the fringes remains constant (= / d).The actual separation of the
fringe increases in proportion to the distance of the screen from the plane of the two slits.
(b) The separation of the fringes (and also angular separation) decrease.
(c) The separation of the fringe (and also angular separation) decreases.
(d) By slightly increasing the width of the slits, we are only increasing the intensity of incident
beam. Again no change in , D, d. so  unchanged but sharpness of the fringe increase.
6. 1.625 mm
7. 0.30 mm
180
8. × 2 × 10–4 degree = 0.011º

DPP – 2
1. (C) 2. (D) 3. (A) 4. (B) 5. (D)
6. (C) 7. (D) 8. 0.9 mm

DPP – 3
1. (D) 2. (C)
3. (a)  = 4.0 × 10–4 m
 2
(b) and
3 3

4. 5. (A)  6. (D) 7. maximum
4(  1)
8. (D)

DPP – 4
1. (A) 2. 1.45 3. 100 nm 4. (B)
5. (i) 1.8 m (ii) 1.2 m
6. (D) 7. (A) 8. (A)

DPP – 5
1. (B) 2. (A) 3. (C) 4. (B) 5. (C)
6. (D) 7. (B) 8. (C)

DPP – 6
1. (D) 2. (A) 3. (A) 4. (C) 5. (B)
6. (B) 7. (B) 8. (D) 9. (C)

DOWNLOAD OUR APP & GET : 353, Rajeev Gandhi Nagar, Instrumentation Limited Colony, Kota, Rajasthan 324005
UNLIMITED PRACTICE FOR FREE
: www.atpstar.com PAGE NO.-187

You might also like